You are on page 1of 160
Bob HE 2008 Nguyén Van Mau (Chui bién) TOAN ROI RAC VA MOT SO VAN DE LIEN QUAN | (Tai ligu béi dudng he 2007) ) HA NOI, 05-14 ‘THANG 8 NAM 2007 ‘Trén bén muci nim thuc hién "Chuong trinh dao tao va bdi hoc sinh nang khiéu toén bac phé thong" 1A mot chang dung ciia mot chu trinh dic bit gan véi su khéi ddu, truéng thinh va ngay cing hodn thién xuét phét tir mét mé hinh do tao ning khiéu Téan hoc dae bigt tai Dai hoc Téng hyp Ha Noi. Huéng dio tao mii nhon niy mang tinh dot phé cao, da ddo tao ra céc thé hé hoc sinh c6 nang khiéu trong linh vc toan hoc, tin hoc va khoa hoc ty nhién: Vat If, Hod hoc, Sinh hoc va khoa hoc sy séng. ‘Trong ditu kign thiéu thén vé vat cht kéo dai qua nhiéu thap ky va trdi qua nhiéu thch thttc, ching ta da tim ra huéng di phi hgp, dé di lén ving chéc va én din, di tim tdi, tich luy kinh nghigm va c6 nhiéu sdng tao dng ghi nhan. Céc thé he Thiy va Trd da dinh hinh va tiép can v6i thé gidi van minh tién tién va khoa hoc hign dai, cap nh&t thong tin, sing tao phuong phép va tap dugt nghién citu. Gan v6i vige tich eye déi méi phutong phép day va hoc, chutong trinh dio tao chuyén Toén dang hung téi xy dug hé théng chuyén dé, dang né lye va da té chtte thinh cong Ky thi Olympic Toan quéc té lin thi 48, nam 2007 tai Viet Nam, dugc ban bé quéc té ca nggi. Sau gan nita thé ky hinh thinh va phét trién, c6 thé néi, gido duc mii nhon phd thong (gido duc nang khiéu) 48 thu duge nhing than tyu rye r6, duge Nha nude dau tur c6 higu qua, duge x4 hoi thita nhan va ban bé quéc té kham phuc. Céc doi tuyén quéc gia tham dit céc ky thi Olympic quéc té o6 bé day thanh tich mang tfnh én dinh va c6 tinh ké thita. Dac biét, nam nay, cdc Doi tuyén Ton vi Tin quéc gia tham dy thi Olympic quéc té da dat duge thanh tich néi bat. Doi tuyén Ton Viet Nam da von len ditng thit ba (theo sy sp xép khong chinh thttc) trong s6 95 doi tuyén cdc nude tham dy IMO48. ‘Tix nhidu nim nay, céc hé nang khiéu Todn hoc va cdc Trung THPT Chuyén thutng sit dung song song céc sich gido khoa dai tra két hap véi sich gido khoa chuyén bigt vA sich chuyén d@ cho cdc H@ THPT Chnyén. Hoc sinh céc lép nang khiéu da tiép thu t6t céc kién thite cd bin theo thi Ivgng hign hank do Bé GD va DT ban han. Hign nay, chuong trinh cai céch gido duc dang buéc vao giai doan hodn chinh bo SGK méi. Thi lugng kién thite eiing nhu trat ty kién thite co ban cé nhiing thay ddi déng ké. Céc kién thite ny dang duge cin nhée dé n6 vn nim trong khuén khé hien hanh cita céc kién thtte nding cao d6i véi céé lp chuyén ton. Vi lé d6, viéc tién hin viét hé théng céc séch chuyén dé cho céc l6p niing khiéu cn duge tién hanh khdn truong va duge xem xét toin dién tir phfa cée chuyén gia giéo duc va céc cd gido, thay giéo dang truc tiép ging day c4c lép chuyén. Duge sit cho phép cita BO GD va DT, Trudng Dai Hoc Khoa Hoe Ty Nhién, DHQGHN phéi hgp cling véi céc chuyén gia, cée nhA khoa hoc, cdc 06 gido, thy gido thudc DHSPHN, DHQG TpHCM, Vien Tosn Hoc, Hi Toan Hoc Ha Noi, Tap Chi Toan Hoc vi Tudi Tré, cée Trung THPT Chuyén, Cae $6 GD va DT... t6 chttc bai dudng cée chuyén d& nghiép vu sau dai hoc nhim béi dudng hoc sinh gidi cdc mén Toén hoc va khéi kién thitc khoa hoc tu nhién nhu 1A mot ta sich dc biet phuc vu di dudng hoc sinh gidi. Ching t6i xin gidi thigu cuén séch cia nhém céc chuyén gia, céc thay gido véi su tham gia dong déo cia céc ding nghiép tham du Trutmg h® 2007 vé chuyen dé "Tan ri rac va mot s6 vin dé lién quan". Cuén séch nay nhiim cung cép mét s6 kién thite chuyén d8 bat ding thtte 6 mie 46 kh6 vé tan rdi rac, dai s6, s6 hoc, va gidi tich. Day cing la chuyén dé va bai gidng m& cée te gid da gidng day cho hoc sinh céc doi tuyén thi Olympic Toén hoc quéc gia va qubc 8. Ching ti cing xin chan thinh cdm on c&c ban doc cho nhiing ¥ kién déng gép dé cudn séch ngay cng hoin chink. ‘Thay mat Ban Té Chite GS TSKH Nguyén Van Mau Ldi néi dau ........ Dé thj to mau va mét sé bai toan khong m4u myc Ding Huy Rud .eeeeseeceeseseeees Logic hinh thie va 4p dung Nguyén Van Mau .... Cong thife tinh s6 phan tit cha Vi Dink Hoa ... hgp cdc tap hop Mang luéi 6 vudng trén mat phing Vii Dinh Hoa ... Nguyén ly Dirichlet va mt s6 bai toan 4p dung Nguyén Duy Thai Son .. Mot sé phudng phap giai c4c bai téan té hop nang cao Dang Hing Théng ... Xay dung song anh gi Hiynh Tén Chau . m6t sé bai todn té hdp Phuong phap thiét lap hé Hiynh Tin Chau ose. seceees shite truy héi trong t6 hdp Y twéng giai va su tudng minh Idi gidi qua mot sé bai téan té hdp Lé Van Quang . Gidi thigu mot sé bai toan dai sé cé6 xudt xit tix hinh hoc Nouyén Dang Phat... Bat bién, don bién va ting dung Trin Nam Ding .. seteeeeeebeee LIT Mot s6 van dé cia Todn rai rac Nguyén Van Tién..... D6 thi to mau va mét sé bai todn khong mau muc Dang Huy Ruan Ly thuyét dé thj néi chung, dac bist dd thi to mau duge van dung dé gidi céc bai todn khong méu mu rat higu qua. Dé gidi toan thong qua dé thi cén thye hign theo hai bué Du tien xay dung dé thi dé mo ta cdc quan he, diéu kign duge phat biéu trong bai tofn. Sau dé cin cit vio céc khang dinh cita ly thuyét dé thi dé suy ra dap An. Trong phan nay chi dé cap dén toén khong méu muc. ic thong qua dé thi t6 mau dé gidi mot sé dang Déi véi méi dang todn khong mdu myc déu dua ra khing dinh tuong img vé dé thi t6 mau dé c6 thé vin dung gidi quyét hang loat bai ton thudc dang dude xét. Dé thi dugc goi la day di, néu mdi c&p dink cia né déu duge néi bing mot canh. Dé thi day di gom n dinh. Dé thi day di gdm 3 dinh (4 dinh) véi céc canh duge to mau bing cing mot mau duge goi 1a tam gic (tit giée) cing mau. 1 Dangi 1.1 Bai toén Bai toan 1 (Dé thi Olympic Toén Quéc té) Mudi bay nha bac hoc viét thu cho nhau. M6i ngudi déu viét thu cho tat cd ngudi Khéc. Cc thut chi trao déi vé 8 dé tai. Timg cap nhit bac hoc chi viét thu trao déi vé ciing mot dé tai. Ching minh ring chi cd it nhét 3 ha béc hoc wiét thu cho nhau trao déi vé cing mét vén dé. Bai ton 2 Trong mot cude giip 9 quéc té cd 17 nha ngoai giao tham gia. Méi ciip nha ngoai giao chi trao déi tryc tiép vdi nhau béing mot trong ba ngon ngi: Anh, Phdp, Ditc. Ching minh ring ludn ludn tim dude ba nha ngoai giao, ma ho c6 thé trao déi truc tiép duge bing mét trong ba ngén ngit ké trén. Bai todn 3 Méi cap déi tugng cho trudc chi cb mét trong ba quan hé: ty, ta, ta Chitng minh réng luon lun tim dude ba déi tugng, ma méi cfip trong b6 ba nay ciing 6 quan he ts (1 2) vdin mau canh (céc canh duge 16 bing n mau), sao cho khong tam gide ciing mau nao, luén ludn 06 hinh 5 canh vdi ede canh ciing mau ua chic dudng chéo duge t8 bing cic mau khéc. Chang mink. Bing quy nep theo n. 1. Co 88 quy nap: Voi n = 2 dé thj tuung tng G2 day di c6 by —1 = 5 dinh va 2 mau canh (xanh, dé) khong c6 dd thi con Ky cing mau. Khi dé Gp c6 thé biéu dién 6 dang hinh 5 canh véi canh cing m&u dé va dutng chéo mau xanh. That vay, do Gp dy di nén méi dinh xuét phat ding 4 canh duge to bing 2 mau. Chinh xéc hon, tai timg dinh, méi miu duge t6 trén diing 2 canh. Gia sit ngudc lai, tai dink A mau d6 duce t0 trén 3 canh 1A AB, AC va AD. Khi d6 mot trong 3 canh BC, BD,CD mau dé, dd thi c6 tam gide 43. Nguoc lai cf 3 canh déu mau xanh, dé thi 6 tam gidc xanh, Nhu vay mu thudn v6i gid thiét, Gid sit tai A 06 cfc canh dé la AB va AC (dung lién nét), cn AD vA AB la xanh, (dudng nét ditt). Khi 46 canh BC phai xanh vi ED la dé (hinh 1) Hinh 1 Hinh 2 Hai canh BE va CE khong thé cing xanh. Gid sit BE dd, thi CE xanh, Suy ra CD dé va BD xanh. Vay ta duge hinh 5 canh v6i canh mau dé va dudng chéo xanh (binh 2). 2. Quy nap: Gid sit khing dinh ding voi n = k. Xét d6 thi Guy, dy di v6i Beegayaa 1 dinh, k+1 mau canh va khong c6 dé thi con Ky cing mau. Méi dinh ciia Gey1 xudt phdt (bi 41 — 1).(k +1) canh voi k +1 mau, nén phai c6 ft ahét bi41— 1 canh cing mau. Gid sit tai dinh A c6 by41 —1 canh cling duge t8 boi mau ™y. Khi dé trong céc dinh d6i cia A khong c6 c&p dink nao duge n6i v6i nhau bai canh mau m, (tréi lai thi c6 Kg cing mau m,). Xét dé thi con day di Gy lap nén tit bey, — 1 dinh déi cia, A c6 canh chi t6 bai & mau (trit mu m,) va khong cé Ka cing mau, nén theo gid thiét quy nap, trong Gy ¢6 hinh 5 canh véi canh cing mot mau va dwéng chéo 8 cde mau khée (tAt c& déu khong 18 mau my). Vay trong Gs 06 didu clin khing dinh. 3. Cid tosn 1°) Xay dung dé thj mo ta quan he Cée dé thi tuong ting véi 3 bai toan da cho duge xay dung nhu sau: a) Dinh: Lay 5 diém tren mat phing, khong c6 3 diém nao thang hang tuong tng v6i 5 thanh vién (5 86 nguyén duong, 5 d6i tuong da chon ra). Ding ngay tén céc thanh vin (cdc 6, tén céc déi tong) dé ghi tren céc diém tung img. b) Canh: Dang - Canh dé dé n6i gitta hai dinh tuong ting v6i hai ngudi quen nhaw (hai s6 6 uée chung, hai déi tuong c6 quan hé ty). ~ Canh xanh dé néi gitta hai dink tuong ting véi hai ngudi khong quen nhaw (hai s6 nguyén t6 cling nhau, hai déi tugng c6 quan hé ty) Dé thi G; (1 <4 < 3) mo tA ton bo quan hé diéu kign dugc cho trong bai toan i, nén trong G; khong ¢6 tam giée cing mau, 2) Suy ra dép én. Theo khang dinh 2 v6i n = 2 dé thi G; Ia da giée 5 canh véi ce canh mau dé va céc dudng chéo mau xanh ho&e nguge Iai. Khi dé dya theo dung gip khic Khép kin mau dé ma sp xép cée thinh vien (céc s6; cdc d6i tutgng) tuong tg ngudi xung quanh mt ban trén (lén mot duéng tron), thi méi thanh vien (mdi s6, méi d6i tuong) sé ngéi gitla hai ngudi ma thinh vién cé quen (dting gia hai s6 ma n6 06 wéc chung; ding giita hai d6i tugng ma né e6 quan he 2). 3 Dang 3 3.1 Bai toan Bai toén 1 Trén méit phiing léy 6 diém thy y, khong c6 3 diém ndo thdng hang va khodng céich gita cdc cip diém khéc nhau tig doi mot. Ching minh ring tén tai tt nhdt mot cp diém ma doan théng néi gitta chiing 1 canh ngdn nhat ciia mot tam gidc nado dé, déng thai la canh dai nhét ciia mét tam gide khéc 6 dinh la cde diém da cho. Bai toan 2 Ching minh ring trong n (n> 6) ngudi ty ¥ ludn chon duige n — 4 66 ba, ma trong méi B69 ba nay hot ting doi mot quen nhau hoiic titng doi mot khong quen mhau. Bai ton 3 Ching minh ring trong n (n > 6) 86 nguyen duong tiy ¥ luén luén chon duge n—4 b6 ba, ma trong méi b6 ba nay ting cap s6 cd ude chung hotic nguyén té cing nhau. Bai toan 4 Chiing minh riing trong n (n > 6) adi twong tiy y lon chon ra duge n— 4 86 ba, ma trong méi bp ba ndy hode titng cp cd quan hé ty hodc tig cap c6 quan hé tz Bai toan 5 Voi n = 5 thi cdc khdng dink phat biéu trong céc bai todn 2, 4 con dring nita khong? Néu cdc khéng dink trén khong ding hay cho phan vi du. 3.2 Khang dinh Khang dinh 3 Dé thi day di gdm n dink (n > 6) va duge t6 bing khong qué 2 mau canh, thi ludn c6 tt nhat n — 4 tam gide cing mau. Chutng minh. Trung hop 1: DB thi diy di G, 06 n dinh (n > 6) duge to bing mot mau canh. Khing dinh dé dang duoc ching minh. Trutng hop 2: Da thi day dii G, c6 n dinh (n > 6) véi 2 mau canh (ching han xanh, dd). Ta phai khing dinh Gy 06 it nhét n - 4 tam giée cing mau. Diéu nay duge ching minh biing quy nap theo s6 dinh cita d8 thi. 1. Ca sé quy nap: n = 6 thi dd thi tuong img G5 day dit v6i 2 mau canh (xanh, 48). ‘Ta phai ching minh Ge o6 it nhét (6 — 4) = 2 tam giée cing mau. Theo dinh lf 2.19, Ge ludn 6 ft nh&t mot tam giée cing mau. Do dé ta phai ching minh trong Gs c6 thém ft nhat mot tam giéc cing mau nifa. Khong mt tinh téng quét, ta goi céc dinh ciia Ge 1 A, B,C, D, E, F va tam giée cing mau la tam giée ABC véi céc canh mau dé (nét lién), (hinh 3). Hinh 3 Ta xét cée trutng hop sau c6 thé xdy ra: 1°) Ci ba canh AD, AE, AF déu duge to mau dé (hinh 4), Khi d6, néu c6 ft nh&t mot trong ba canh DE, EF, DF dé thi trong Gs o6 thém st nhét mot tam gide cing mau nia (tam giéc dé) Nguge Iai, néu c& ba canh DE, EF, DF xanh (nét ditt) thi trong Ge c6 tam giée cing mau thit hai (tam giée xanh). 2%) CA ba canh AD, AB, AF du dige t6 mau xanh (hinh 5) Ching minh tuang ty trudng hop 1°). 10 Minh 5 Hinh 6 Néu c6 ft nhdt mot trong ba canh DE, EF, DF xanh thi trong Ge c6 them ft nhit mot tam giéc nia cing mau (tam giée xanh). Neuige lai, c& ba canh DE, EF, DF dé thi trong Go c6 tam giéc cing mau thi hai (tam giée 43 3°) Trong ba canh AD, AE, AF c6 hai canh dd, ching hen AD, AE dugc to mau dé (hinb 6) Khi dé; néu 6 St nh&t mot trong ba canh CD, DB, CE dé thi trong Gs c6 them it nh&t mt tam giéc nifa cing mau (tam gide dd) Neue lei, cf ba canh CD, DE, CE xanh thi trong Ge 06 tam gide cling mau thit hai (tam giée xanh). 4°) Trong ba canh AD, AE, AF c6 ding mot canh dé, ching han AD duge t6 mau 6 (binh 7). Hinh 7 Hinh 8 a) Mot trong hai canh BD,CD dé. Khi dé trong Ge c6 tam giée ching mau tht hai (tam gide da). b) Ca hai canh BD, CD xanh (hinh 8). by) BF xanh: Khi dé trong Gy 06 tam gie cing mau thit hai (tam giée AEF xanh). ba) EF dé: bai) BE dé (hinh 9) bait) Ho&e BF hoje CE dé, ta cé tam giée cling mau thit hai (tam giéc 8). bei2) C& hai canh BF vi CE xanh (hinh 10) bei21) Hote DE hote DF xanh, ta c6 tam giée cling mau thtt hai (tam giée xanh) uw Hinh 9 bai22) Ca hai canh DE va DF déu dé, ta cé tam gidc cing mau thit hai (tam giée DEF 4) ba2) BE xanh (hinh 11) Hinh 11 Hinh 12 bz21) DE xanh, ta c6 tam gidc cling mau thit hai (tam giée BDE xanh). bo22) DE dé (hinh 12). be221) DF dd, khi dé ta c6 tam giée cing mau thit hai (tam giée DEF dé) bo222) DF xanh (hinh 13). bz2221) Hoke BF hoje CF xanh, ta cé tam gidc cng mau thtt hai (tam gide xanh). b22222) Cé BF va CF déu dé, ta c6 tam gic cing mau thtt hai (tam gidc BOF 8). ‘Vay trong moi trudng hop, Ge déu c6 it nbAt hai tam gide cing mau. 2. Quy nap: Gid sit dink ly ding voi n = k. Xét dd thi Guys dy dii voi (4+ 1) din, dugc t6 bing 2 mau xanh, dé. Ta phai ching minh Gx4. 06 it nat (k-+1—4) = (k-3) tam giée cling mau. That vay, vi Gis day dit gdm (k +1) dinh (k+1> 7) voi hai mau canh nén o6 ft nhét mot tam gide cing mau. Gia sit céc dink cha Guy 1A Ay, Aa,...,Aeyi va tam giée cing mau IA tam giée AjA2As (ching han mau dd) (hinh 14). Loai Ai va cé canh xudt phét tir Ay ra khdi dd thi Gey1, ta c6 dd thi Gy voi hai mau canh (xanh, 46). Nen theo gid thiét quy nap, trong Gy luon o6 it nhét (k— 4) tam side cing mau. 12 Hinh 13 Hinh 14 Khoi phue dinh Ai cling céc canh thude A, ta duge 48 thi Gii2 véi hai mau canh (xanh, 48) va Gey 06 St nbat (k-4+ 1) hay (k 3) canh cing mau. Khing dinh duge chiing minh. 3.3 Giai toan Giai bai todn 1. Xét t&t cd céc tam gide c6 dinh 18 cée diém da cho. Vi khoaing céch giiia cc cap diém da cho khée nhau ting doi mot, nén méi tam gidc c6 dinh IA cée diém a cho déu 6 canh ngfn nbat va canh dai nhét. Déi véi méi tam giéc nay ta ding mau xanh dé to canh ngfn nhét. Sau khi tat cA cic doan thang n6i giita cdc c&p diém da cho duge phép t6 mau xanh da to xong, phan doan thing cin lai to mau dé. Dé thi G nhan duge 18 dé thi day di gdm 6 dinh v6i céc canh duge tO bing 2 mau (xanh, dé), nén theo khang dinh 3 trong G cé it nh&t hai tam giéc cing mau. Vi tam gide mao efing c6 canh ngin nhAt duge to mau xanh truée, nén cée tam giée cing mau déu li tam giée xanh. Khi d6 canh dai nht trong méi tam gide nay 1A doan thing cin tim. Béi vi trong tam gidc ta xét n6é déng vai trd canh dai nh&t, nhung vi doan thing nay c6 mau xanh, nén n6 da 1A canh ngén nh&t cia mot tam gid ndo dé trong céc tam. giéc c6 dinh 1A céc diém da cho. Gidi bai toan 2-3. 1°) Xay dyng dé thi mé té quan hé. Céc dé thi tuong ting v6i céc bai ton 2-4 duge xéy dung nhu sau: a) Dinh: Lay n diém (n > 6) tuong tng véi n ngudi (n sé nguyén, n d6i tugng) da. chon ra. Diing ngay tén ngudi (cdc 86, ky hiéu cdc déi tugng) dé ghi trén cdc diém tung ing. b) Canh: Ding - Canh dé dé néi gitta hai diém tuong tng véi hai ngudi quen (hai s6 6 wéc chung, hai d6i tugng c6 quan hé t1). ~ Canh xanh dé n6i gia hai diém tuong (mg véi hai ngudi khong quen nhau (hai s6 nguyén t6 ching nhau, hai d6i tugng c6 quan hé ta) Dé thi G (2 1) doi mot nguyén t6 cing nhau. Héi trong céc sé thudc {1,2,...,n} (n > 1) 06 bao nhiéu sé khong chia hét cho bat crt sé nao thudc {a1,a2,..., Am! Gi Nhén xét ring s6 céc sé nguyén duong khong vugt qué n chia hét cho sé (1 1) khong chia hét cho b&t ott sé ndo thuge {a1,02,-.-,4m} tinh bling cong thiic ~ (+++) + (2a) G++ Ete) ee 4102" Ory. Nhan xét ring, khi céc tinh chat duge ligt ké nhut trong Dinh ly Silvester khong c6 tinh doc lap thi céng thttc tinh todn phai thay ddi. Ta xét vi du sau day. Bai toan 4 Héi trong tap A = {1,2,...,280} c6 bao nhiéu sé khong chia hét cho 2, 3, 4,5, 6,7. Giai, Nhan xét ring, s6 céc s6 trong A khong chia hét cho mt trong céc s6 2, 3, 4,5, 6,7 cing bing mét s6 céc sé trong A kh6ng chia hét cho mét trong céc s6 2, 3, 5, 7. Ta xét xem trong tap A c6 bao nhiéu s6 chia hét cho mot trong cdc 6 2, 3, 5, 7. Ki higu A, 18 56 céc 86 thudc A chia hét cho 2, Ay 18 86 cée s6 thudc A chia hét cho 3, Ag la 86 cdc s6 thude A chia hét cho 5 va Ay 1 86 céc s6 thud A chia hét cho 7. Khi d6 A, U ApU Ag U Ag Ia tp céc 86 chia hét cho mot trong céc s6 2, 3, 5, 7. Ta c6 = [=| = 280 _ 40, va 22 [Arn Add [2 0; 1420 al = [22] = 6 | [42 Ad = [>| = 15; [As Ad = [FI | va 280 1A. Agn As = ES = 9141.42 Ay] 280 ‘0 ain Asnad = [ ] = Vapy == F Vosby..wd Ching minh. Phuong phép ching minh hoin toan nhu d6i véi Dinh ly Sylvester khi ta gin méi déi tuong cing mét gié tri bang 1, 23 Cong thic tinh sé phan tw cia mét hgp cac tap hop Vi Dinh Hoa ‘Trong cudc s6ng nhiéu khi xudt hign nhiing bai toén phai tinh s6 lugng phan tit cha mot tép hop thong qua nhitng tap hgp con cita ching. Ching han: ‘Vi du 1 Trong mét bai kiém tra todn 06 hai bai todn. Trong ca lép c6 30 em lam duge bai thet nhét va 20 em lam duoc bai thit hai. Chi cé 10 em lam duac c& hai bai todn kiém tra. Hay tinh 86 hoc sinh trong lép. Goi Aa tap hop hoe sinh gidi duge bai ton thi nhét, va B la tap hap hco sinh gidi duge bai toén thit hai, thi ANB IA tap hop hoc sinh gidi dugc o& hai bai ton. Bai tofn dugo dat ra 1a phai tinh s6 phan ti cia AU B. Néu A va B la hai tap hop rdi nhau, thi thy dé dang ring: |ANBl=|Al +B]. ‘Trong trutng hop A va B c6 giao khéc réng thi ding thite trén khong cdn ding nita, ma ta s8 06 cong thie [AUB =|Al +|B]-|An Bl, bai vi trong téng |A| + |B] thi cée phin tit chung trong |AN BI (cia cd A va B) duge tinh lap déng hai lan. Sit dung cong thitc nay, ta thy s6 hoc sinh cita lép trong vi dy trén cba ta 1a 30 +20 — 10 = 40 em. Nhiéu khi, bai ton ta g&p trd nén phitc tap hon khi phi tinh s6 phan ti cha mot tap hgp c6 nhiéu hon hai tap hgp. Vi du 2 Lop 12A phdi lam mét bai kiém tra Todn gém cé ba bai todn. Biét ring méi em trong lép déu lam duoc it mhét mot bai, trong lép cé 20 em lam dusge bai todn thit nhét, 14 em gidi dude bai todn thit hai, 10 em gidi duge bai todn thit ba, 6 em gidi dugc cd hai bai todn thit nhét va thit ba, 5 em gidi dude cé hai bai thit hai va thit ba, 2 em gidi duoc cé hai bai thit nhdt va thi hai, vad cd mot em dude 10 diém vi da gidi duoc cd ba bai todn. Héi ring lép hoc 06 bao nhiéu em tat ci? Goi A la tap hop cée em hoc sinh gidi dugc bai toan thit nh4t, B 1A tap hgp cée em hoc sinh gidi duge bai todn thit hai va C la tap hyp cae em hoe sinh gidi duge bai ton 24 thit ba, ta phdi tinh s6 phn tit cia tap hyp AU BUC. Trong nhigu trugng hop khée, chiing ta phai tinh s6 phan tit cia mot hgp gém nhiéu tép hop con, va phan nhiéu cée bai todn nay 1a céc bai toan khé véi hoc sinh khong hé duge hoc cong thie tinh té hop. 1 Xé€p cé lap véi tan sé cho truéc Mot phn tit a; ctia mot day k phin tit cho truéc aia2...a, dutge n6i la c6 tin s6 lip ky, néu nhu n6 xudt hign trong day ding &; lin. Trong myc nay ching ta xét bai todn sau: Bai toan. Cho trudc mét tap hop A = {ay;a9;...; an} vd k la mot s6 duong nguyén cing wai cde 36 ty nhién ky, kay... hn théa man D7 ky = k. Hay tinh s6 P(k, ko,...,hn) cc day k phan ti ciia tap hop A sao cho phan tit a; cia A cé tan sé Idp la ki cho moii 1 cho moi gid trid 1 ta 06 (ety)? = Docherty! io Ching mink. Dé khai trién (x + y)", ta thuc hign phép khai trién lay thita mot céch hinh thc ma khéng rit gon ching. Ching han (ety) =crt sy tyct yy. Nhu vay ta 06 thé biéu dién hinh thie (e+uy (z+y)(z+y) (ey) = Dreier. cas 6 d6 c6 t&t cA 2" s6 hang cicp...cy voi c: € {2;y}. Trong khi thyc hién phép rat gon, ta phai dem nhém t&t cA céc s6 hang c6 cing s6 mi cia z va y lai voi nhau. Véi méi O 2 tap hop tay §. IALUA2U++-UAnl = SOIAd— SO A Aste ra Lign met (HPAL ARN An > An; Angi. Luu § ring Bay gid xét n+ 1 tap hgp tay y Ai, Aa. (A1UA,U “UAR) 9 Anas = (4s Ana) (™) i= Cho nén ta 06 |(AyU A2U---UAn) Angi! = Uf; (4s Ansi)]- Sit dung (*) cho vé phi cia (*), ta thu duge ALU---U An) A Ang] YAN veal — D IANA Anat Liga + SANA AN Ana 1 iGiek on mk (1) JALN Ag AR A Ansa Dit A= A,UAdU:--UA, va B = Anyi va 6p dung ding thie |AUB| = |A]+{B|—|AnB| (trudng hgp n = 2), ta thu duge diéu cn chting minh att IALUUAnU Ansa] = 1A SO A Ase ro 1s mot (HTP ALA Aa An Anal ‘Vay dinh ly duge’chting minh cho trutng hop n+ 1. Dinh ly duge chiing minh. Céch 2: Ta xét mot phan tit a € AyUA2U---U A, bit ky. Gid sit ring a thudc vio r (1 3 diém thi khi néi c&c diém voi nhau béi cdc doan thing cé dinh Ja diém trong n diém da cho thi ta thu duge mot da giée 18i hoe mot doan thing chita céc diém cdn lai 6 bén trong, va ta goi n6 (da gide 18i ho&c doan thing nay) 1a bao Iéi cia tap hgp n diém nay. R&t nhiéu bai todn cia If thuyét té hgp c6 thé gidi bing c&ch van dung bao Idi cha céc hinh hodc cia tap hgp diém cho truéc. Vi dy 1 Trén mat phang cho truéc mot 6 diém khong cing ndm trén mét dudng thang. Chiing minh ring ton tai ba diém sao cho dudng tron di qua ba diém nay khong chia diém nao cho trude d bén trong. ‘Ta xét bao léi cia tap hyp cac diém cho truéc nay. Do c6 ba trong s6 céc diém nay Khong thing hang, cho nén bao ldi cite chting IA mot da. giée di. Xét mot canh AB cha da giéc Ii nay. Trong nhiing dinh cdn Iai, gid sit C 1 dinh nhin canh AB véi mot goc Jon nh&t. Khi dé dudng tron ngogi tiép tam giée ABC khOng chita diém nao da cho bén trong. Ngo&i tam giéc Ia da giéc o6 3 dinh ludn 1a hinh 18i, mOt n-gide bat ld voi n > 4 c6 thé khong phai la da giéc Idi. Mot da gidc duge goi 18 da gide cd canh khong ty cit néu anh cdc canh cha n6 déi mét khong c&t nhau triy nhing canh lién tiép c6 thé c6 dau mit chung. Moi da gidc Idi cing la da gidc c6 canh khong tut c&t. Cho truéc mot da giéc cé canh khéng ty cét, ching ta luén c6 thé chia n6 thanh cfc hinh Idi, cy thé JA chia né thAnh cdc tam gidc, béi cdc dutng chéo khong c&t nhau trong da giéc. Dé ching minh didu nay, truée hét ta chimg minh két qua sau: inh If 1 Trong mot de gidc 6 canh khéng ty cit lun cé mét dudng chéo nim hoan toan trong né. ‘That vay, néu da gide da cho IA da gidc Idi thi khiing dinh cia bai todn 1a hién nbién. Neuge Iai, néu da gide da cho khong phai lA da giéc 16i thi n6 c6 mot dinh, goi la A, 30 6 gc 16n hon 180°. Tir dinh A nay ta ké mot tia Ax ndo dé, cit bién cia da gide tai mot diém M. Do A > 180°, cho nén c6 mt trong hai canh bén cia A, ching han AB, sao cho tia AB (di tir A tdi B) khong c&t canh cia da gidc c6 chita diém M. Khi ta cho tia Az quay quanh A theo chiéu tir vi tri ban dau t6i vj tri AB, tht diém M chay tren bién ciia da giée va lic nao dé né phi di chuyén tit mOt canh nay sang canh khéc cita da gidc d& cho. Tai thdi diém d6, tia Az phai di qua mot dinh C no d6 khée A ca da giéc. Lite dé dutng chéo AC chinh IA dung chéo nim hoan toAn trong da giéc. Dinh If sau day hién nhién 18 hé qua cba dinh If trén: Dinh li 2 Mét n-gide cé canh khéng ty cét ludn cé thé chia thank n — 2 tam gide béi cde dudng chéo khéng ct nhau ndm hoan toan trong da gidc. Ta ching minh khdng dinh bing quy nap theo s6 dinh cia da giée. Voi n = 3 khang dinh cia dinh If hién nhién ding. Gia sit két lun cla dinh If ding cho moi k-giée (k 3 vak-+é=n-+3. Theo gid thiét quy nap thi céc k-giée va é-gidc nay o6 thé chia thanh k — 2 va €~2 tam giée bai céc dudng chéo khong cét nhau va hon ton nim trong chiing. Bang céch d6, ta da chia (n+ 1)-giée da cho thanh k+€—4=n—1 tam giée bai céc dung chéo khong c&t nhau va hoan toan n&im trong (n+ 1)-giée da cho. Dinh If duge chitg minh cho trugng hgp (n+ 1)-gide. ‘Ta goi mot tam gidc c6 céc dinh 1a dinh luéi cia mang hi6i 6 vuong 1A tam giée don néu nhut n6 khong chifa m6t dinh hi6i ndo khéc trén canh hofe bén trong tam giéc. Dinh If 3 Céc dinh cia mot k-gide cb canh khong ty cit F (khong nhét thiét phdi 16i) nam é cae diém nguyén. Bén trong né cén diém nguyén, con trén bién m diém nguyén. Ta néi che diém nguyén lai vdi nhau bing cdc dogn thdng khdng cit nhau sao cho tét cd céc tam gide thu duge la tam gide nguyén, khi dé s6 tam gide don thu duce bing nhau va bing ding 2n +m —2. Goi s la 86 cdc tam gid don thu duge. Téng cdc géc cia s tam gide don nay 1a 1 x s. ‘Téng nay bang diing téng cc géc quanh céc diém 6 bén trong F’ va céc diém trén chu vi cia F (gdm céc dinh cia F va céc diém nguyén 4 trén bién cilia n6). Téng cdc goc quanh céc dinh cia F bing 7 x (k — 2) (do F c6 thé chia thanh & — 2 tam giée bai céc dudng chéo khong ty cét 4 hoan to&n bén trong né theo dinh If trén). Teng cdc g6c quanh méi diém nguyén 6 trén canh cia F 1a 7, va téng cdc géc quanh méi dinh trong F 1a 2r. Do dé ta c6 ding thite: wx (k—2) 40x (m—k) +2nxn=ax s, ‘Va ta thu duge dang thite 2n +m — Bai tap 31 1. Ching minh ring mot da gide IA da giée Idi khi va chi khi b6n dinh bit 1d cha ching tao thinh mot tit gide Idi. 2. Trén m&t phiing cho truéc nam diém sao cho khong cé 3 diém nao trong ching ~ thing hing. Ching minh ring c6 thé chon ra trong ching 4 diém la dinh cia mot tit gide Idi. 3, ‘Tren m&t phing cho truée n > 5 diém trong d6 khong c6 3 diém ndo thang hing. Ching minh ring tén tai it nhdt C?_, tit gide 18i ma céc dinh cia chting 18 4 diém trong s6 n diém da cho. (Ki thi Todn quéc té lan thi 11, n&m 1969) 4, Trén mit phng cho mot s6 hitu han diém. Ching minh rang luén tim duge mot diém sao cho gan né nhat c6é khong qué 3 da cho. 5. Tren mat ban dat n hinh vudng bing cét-tong van hinh vudng bing nhya sao cho céc hinh vudng cing loai biing cét-tong (cing nhu cfc hinh vudng cing bing nhya) doi mot khong c6 diém chung. Sau khi xép ching lén mat ban, ngudi ta nhan xét thay ring tap dinh hinh vudng bang nhua hon toan tring véi tap dinh hinh vudng bing cét-tong. Hay ching minh ring cdc hinh vuong bing cét-tong dutge dat tring hoan toin v6i céc hinh vudng bing nhva. 2 Mang lwéi dinh 6 vuong M6t hé théng vo han 6 vuéng tao trén mat phing giéng nhit ta lat gach mot cdi san bdi nhitng vien gach lét 0 vudng dutge goi A mang Iudi dinh 6 vudng. Céc 6 vuong nay duge g0i 18 cée 6 vudng co s8. Cac dinh 6 vudng chinh la céc diém nguyén (diém c6 cA tung do lan hoanh do 1a cdc s6 nguyén) cita mot hé truc toa dd song song v6i cdc canh cia céc hinh vudng co sé va c6 don vi géc 1a do dai canh hinh vudng co 86. Mot da gidc c6 dinh 1a céc dinh Iuéi cia mang 6 vudng dude goi la da gide nguyen. ‘Véi mang lu6i dinh 6 vudng cé nhiéu bai todn khé thi vi. Sau day 1a mot tinh chét co ban ciia mang lu6i dinh 6 vung. Dinh If 4 Da gide déu duy nhét cb dinh tai céc didn ludi 6 vudng [a hink vwdng Dé thdy rang ta cé thé dung dugc nhitg hinh vudng c6 dinh tai céc diém luéi 6 vudng, ching han hinh vu6ng co sd cia mang luéi 6 vudng 1a mot hinh vudng nhu vay. Bay gid ta chimg minh ring ngoai hinh vudng, ta khong thé dyng mot da giée déu nao khée ¢6 dinh tai céc diém ludi cia mang ludi 0 vudng nao ca. Ta chting minh bing phwong phdp phan ching. ‘Trude hét ta ching minh ring khong thé dung duge tam giée déu c6 dinh tai cdc diém ]u6i 0 vudng. That vay, gid stt dung duge tam giée déu canh a c6 dinh tai céc diém lu6i @ xis a mot 56 v6 ti, do a? 1 mot s6 nguyén. Mat khéc, din tich ota tam giée c6 ba inh I diém lui cba mang luéi 6 vudng 18 mot s6 hitu ti, v6 Ii. Vay, khong thé dumg duge tam gide déu c6 cha mang lu6i 0 vudng. Khi dé dign tich cia tam gide déu ny 18 32 dink tai céc diém ludi cia mang Iuéi 6 vuéng. Bay gid ta chting minh ring khéng thé dung duge nga gidc dau c6 dinh tai céc diém luéi cia hinh vudng, That vay, gid sit khiing dinh nay khong ding, va tén tai ngi gide déu c6 dinh tai diém luéi cia mang 6 vudng, dé gon ta goi dé 1& nga gide dau tét. Xét tap hop: A= {a?:a 18 canh cia nga gide ddu tét}. Do A la mét tap con khéc réng cia tép hop céc s6 nguyén, cho nén tdn tai mot phan tinhé nbét a cia A. Xét nim dinh cia ngii gide du canh a nay, ‘Ta, dem néi céc dung chéo cita ngit giéc lai. D8 théy ring méi dutng chéo cia mot ngii giéc déu song song véi canh déi dign ciia n6. Do d6 hai dudng chéo thy ¢ cia nga sie cling v6i hai canh d6i dién cia chiing lap thinh mét hinh binh hanh, cho nén giao diém cita hai dudng chéo cing 1A mot diém cia mang luGi 6 vudng. Dé chting minh diéu nay ta lap mot hé toa d6 6 truc song song véi canh ciia 6 vudng co sé vi c6 do dai bing 6 dai ciia canh hinh vudng co sd. Khi d6 céc dinh cita mang It6i 6 vudng cd 8 chinh la céc diém 6 toa dé nguyén. Ban doc dé dang chting minh ring giao diém nay c6 toa do bing téng hai toa dé cia hai dinh cuéi cia hai canh di dign trit di toa d6 cia dinh chung cia hai canh nay. Nhu vay nim giao diém cita cée dutng chéo lap nén mot hinh ngii giée déu cé nim dinh 1a nam diém nguyén. Hinh 1: Ngii gide déu DE théy ring ngi gide déu nay cé canh nhd hon a, mau thudn v6i gid thiét ring trong céc ngii gidc déu c6 nam dinh 1a cdc diém nguyén, nga giée déu cé canh nhé nhét Tia. Mau thudn nay chting té ring khong tén tai ngii gidc déu nao c6 cd nim dinh 1 nm diém nguyén. Mat khéc, ta c6 thé théy ring khong thé dung duge Iye giéc déu c6 dinh tai mang luéi dinh mang Iuéi 6 vudng, vi néu cé mot luc giée déu nhu vay, thi cing 6 mot tam gide du (duge tao bdi ba dinh doi mot khong ké nhau cia lye giée déu nay) c6 dink lA dinh luéi ciia mang 6 vudng da cho. Bay gid ta ching minh ring véi n > 7 thi khong thé dung dugc n-giée déu cé dinh tai dinh ludi mang 6 vudng. That vay, gid sit dumg duge mot n-gide déu AyAy--+ A, 06 dinh tai céc dinh cia mang lu6i 6 vudng. Khi dé ban kinh R cia dudng tron ngoai tiép n-gide J6n hon can a ciba n-giée déu A)Ay--- Ay. Ta chon mOt dink O cia mang lu6i 0 vudng va chon céc dinh By, B2--- By sao cho OB;=AiAis1 (Aner = Aj). Vi cde dinh A; va O 1 dinh lugi 6 vudng, cho nén céc diém B, cling 1a dinh IuGi 6 vudng, Ngo&i ra, ta thay 33. dé dang lA B,Ba--+ B, 18 mot n-gidc déu, déng dang véi n-giéc déu A; Ag-+- A, theo hé s6 £ <1. Cit tién hanh nhu vay, sau mot s6 k buée hitu han dit Ién, ta thu duge mot negidc déu c6 canh nhé hon canh cia 6 vudng co sd 1a diéu vo li. Do d6 khéng thé ding duge n-giée déu (v6i n > 7) c6 dinh tai dinh lui cla mang 6 vudng. Bai tap 1. Tim bén kinh 16n nhét cia dutng tron chi di qua céc dinh cia mang luéi 6 vudng ma khong ct canh 6 vudng nao é diém trong cila nd. 2. Cho truéc mét s6 dinh cia mang luéi 6 vudng, ching minh ring ta c6 thé to mau céc dinh ndy béi hai mau xanh va dé sao cho trén méi duéng nim ngang va trén méi duttng thing diing s6 diém duge to dé va s6 diém duge td xanh xAp 24 nhau. (D8 thi Tosn quéc té nim 1986) 3. Chiimg minh rang néu di doc theo céc canh ciia 6 vudng co sé tit mot dinh bat ki i ta tré vé dinh ban dau sau hitu han bude (c6 do dai bing canh hinh 6 vung co 8d), thi sé bude di cilia ta sé la mot s6 chin. vao mdi 6 vudng cd sé mét sé nguyén sao cho méi sé nay bang ding trung binh cong cia bén s6 8 bén 6 vung cd s8 c6 canh ké véi n6. Hay chting minh rin, a) Tit 8 céc s6 duge dién bang nhau, néu nhu né bi chan. b) C6 mot cach dign sao cho cde s6 duge dién khong nhdt thiét phai bing nhau tht cB. 5, Trén ban cd vo han ta thyc hign mOt trd choi nhu sau: ‘Dau tién xép n? quan vao mot hinh vudng gm n x n 6 vudng canh lién canh sao cho trong méi 6 vudng ciia hinh vung chita mgt quan c’. Céch di trong trd chai 1a quan cd chi dutge nhdy theo mot chidu ngang hoc chiéu doc qua mot 0 c6 chtta quan cd d ngay sét bén canh sang mot bén mot O tréng tiép ngay sau a6. Khi d6 quan c¥ 6 6 bj nhdy qua sé bi loai b3. Tim céc gid tri cla n dé c6 thé két thic trd chai sao cho trén ban cd chi cdn lai ding mot quan cd. (Ky thi Toén quéc té néim 1993) 3 Dinh li Picard Mot img dung khé thiét thye cia mang ludi 6 vuong Ia img dung vao viée tinh dign tich ofa céc hinh phang. Ta 6 thé thy hign vigc tinh dign tich cia mot hinh phing bing céch dem phi né béi mot ludi 6 vubng di nhé ri tinh dign tich hinh da cho. Co 86 cho vige tinh dign tich da giée nguyén (da gide o6 dinh 1A céc diém toa d@ nguyén) 18 vige xéc dinh dign tich cia cfc hink tam gidc cé dinh la cfc dinh luéi (con goi la dink nguyén) ma khong chia dinh nguyén ndo khéc bén trong hode trén canh cia né (tam giée don), 34 Dinh Ii 5 Dien tich ciia tam gide don trén mang ludi 6 vudng don vi diing bang }. ‘Truéc tién ta théy dign tich cla mot tam giée nguyén khong nhd hon } Vita cé thé ngi tiép n6 trong mot hinh chit nhat béi céc canh di qua dink né va song song véi céc dudng thing cia mang luéi 6 vuong. Khi d6 dién tich cite tam gide nguyén bing téng hokc higu cia dign tich cia céc hinh chit nhét con va cée tam giée vudng ¢6 canh la 56 nguyén duge tao thanh, nén c6 dang £, véi k la mot s6 nguyén duong, tic IA it nhét cing phai a 3. Mat khdc gid sit hinh chit nhat phi tam giée don ABC cho truéc theo céch trén c6 dé dai hai canh 18 m van sé c6 dign tich bling m xn va chita (m—1)(n—1) diém nguyen bén trong va 2(n +m) diém nguyén trén bién. Ta chia hinh chif nhat nay ra thnh cdc tam gic don béi cic doan thing néi cdc diém nguyén, bit dau 1A eée canh cha tam gidc ABC, va thu duge nu dinh If 3 ca myc truéc cho ta 1 2(m —1)(n~1) +2(m-+n) —2 = 2mn tam giée don, Do dign tich ciia méi tam giée don (ciing I tam giée nguyén) khong nhé hon }, cho nén dign tich ola hinh chif nhat phi n6é khong nhé hon mn. Do é day xy ra ding thiic, nén dign t{ch cia mdi tam giée don thinh phan, dac biét IA dién tich cia tam giée don ABC, cing bing ding } Dinh ls Picard sau day cho ta céch tinh dign tich cia eée da giée nguyén 6 canh Khong ty c Dinh li 6 Cac dink cia mot da gide F ob cank khong ty cit (khong nhdt thiét phi 101) ndm 4 céc diém nguyén. Bén trong n6 cé n diém nguyen, con trén bién m diém nguyén. Khi dé dign tich ctia né bing Span4% ‘Theo dinh If 3 da chting minh 6 muc trudc, ta c6 thé chia F thinh 2n +m —2 tam gide don béi céc doan thing n6i céc diém nguyén nim trong hote trén bien ciia n6. ‘Theo dinh If da chimg minh 6 trén, dign tich otia méi tam gide don chinh bing }, nén dign tich cia F ding bing n+ Sr 2 -1 Bai tap 1. Céc dinh cia mot tam giée nguyén ABC khong chita mot diém nguyén nao trén canh ngoai céc dinh cia n6 va chita ding mot diém nguyén 6 trong tam gidc. Ching minh ring diém nguyén 6 trong tam giéc nay IA trong tam cia tam ide, 2. Tren mat. phang ly nim diém nguyén phan biét. Chimg minh ring tdn tai trong ching hai diém nguyén sao cho doan thing néi hai diém nguyén ndy di qua mot diém nguyén nao d6. 3. Chiing minh ring trong mét hinh ngit gic 1éi nguyén luon cé it mht mot diém nguyén 1a diém trong, 35 4, Hay tinh dign tich nhé nhét ma mot ngii gidc 1éi c6 dinh 1a céc diém nguyén cé thé 06. 5. Trén mét t8 gidy cé ké mang ludi 6 vudng c6 n 6 bj tO den. Chiig minh ring cé thé cit tir to gify nay ra mét s6 hitu han céc hinh vuéng théa mn déng thdi hai digu kign: a) Tat c& céc 6 den déu nim trong cdc hinh vuéng nay, b) Tile dign tich bi to den va dign tfch toin phan trong méi hinh vudng nay & 4 gitta : waz Nguyén ly Dirichlet vA mét sé bai toan 4p dung Nguyén Duy Thai Son Nguyén ly Dirichlet (thuat ngit tiéng Anh: the pigeonhole principle, ciing ¢6 noi goi 23 the drawer principle) - 6 dang don giin nhit - duoc phét biéu dau tién béi G.Lejeune Dirichlet (1805-1859), mot nha ton hoc Ditc gb¢ Php, nhu sau: "Néu nh6t n+ 1 con thé vao n cAi chudng (n € N*) thi ludn 6 (ft nhét 18) hai con thé bi nhét trong cing mot chudng" Mot céch téng quét, ta cé nguyén lf Dirichlet mé rong: "Néu nhét m con thé vao n céi chudng (m,n € N*) thi ludn tn tai mot chudng chita ‘m1 fe nhét 1a1+ [=] con tho". n 6 day, ky higu [a] dutge ding d@ chi phin nguyén ciia s6 thuc a tite 1 s6 nguyén 1én nhét khOng vust qué a. Dang phung phép phan chting, ta cé thé dua ra m6t céch ching minh khé ngéin gon cho nguyen ly Dirichlet (ngay c& duéi dang mé rong); hoc sinh THPT cing c6 thé lam duge vige nay; va didu d6 khong hé lam gidm di gié tri cia ban than nguyen ly. Nguyen ly Dirichlet c6 rét nhiu img dung (higu qué dén bat ngd): sit dung n6, ta c6 thé ching minh duge nhiéu két qua sdu sxe ciia toan hoe. Chinh vi vay, tai céc cude thi hoc sinh gidi toan (quéc gia va quéc 8), nguyén ly Dirichlet thutng xuyén duge khai théc. Dé minh hoa, duéi day, ta xét mot s6 bai toén cu thé. Bai 1 (Putnam 1993). Cho mot day s6 gdm 19 s6 nguyén duong khong vuot qué 93 ua mét day s6 gdm 93 56 nguyén dutong khong vugt qué 19. Ching minh ring tw hai day sb 6 ta 06 thé lan lugt trich ra hai day con cé téng cic s6 hang la bing nhau. Gi . Ta xét bai todn téng quét: Cho hai day (hit han) cée s6 nguyén dung: MS S-StmSn, miyws-Smsm Khi d6, tan tai cfc "chi s6"1 7 x 256 nén theo nguyén ly Dirichlet o6 tam hoc sinh (khéc nhau) Aj, Aa).--,Ag, mA bai Lim ciia ho thi tmg v6i cée bd thude cing’mét tap con Ando 6 (trong s6 256 tap con n6i trén). Nhung 2000 — 8 = 1992 > 7 x 256, nén lai theo nguyén If Dirichlet - tdn tai tém hge sinh (khée nhau, trong s6 1992 hoc sinh cdn lai), 18 Bh, By,..., Bs, 06 bai lam ting voi cée bo thudc cling mot tap con B ndo dé (trong s6 256 tap con n6i trén). Cudi cing, vin c6 1992 — 8 = 1984 > 7 x 256 nén ding nguyén If Dirichlet mot lin nia, ti 1984 hoc sinh cdn lai, ta tiép tuc tim ra tam hoc sinh (khéc nhau), 1& C,,C2,...,Cs, ma bai lam cia ho img véi céc b6 trong cing mot tap con C ndo d6. Tit bén hoe sinh bat ky trong s6 24 hoc sinh Ai, Aa, +) As; Br, Bay. Bay Cry Cay. +s Cay ta Iudn chon ra dude hai hoc sinh 6 bai lam ting véi cdc bé thudc cing mot top con (mét trong ba tap con A,B ho&c C); gid sit, ching han, 46 1a c&c hoc sinh Aj v& A; (i,j €Z; 1 $i 8 x 3 theo nguyén If Dirichlet, cit 25 hoc sinh thi tim duge bén hoe sinh (trong s6 25 hoc sinh nay) c6 bai lam ting véi bén bO z,y,z,t € D C S doi mot khée nhau; vA theo (3), hai hoc sinh ndo trong s6 bén hoc sinh dé cfing c6 bai lm khéc nhau 6 ft nhét 18 hai cau héi, ding nhu yeu cdu cla bai toan. Tu 1/ va 2/, ta thay: 86 tut nhién bé nhét phai tim 1A n = 25. 41 Nguyén ly Dirichlet va m6t sé bai toan Aap dung Nguyén Duy Thai Son Nguyén lf Dirichlet (thuat ngit tiéng Anb: the pigeonhole principle, cling c6 noi goi 1a the drawer principle) - 6 dang don gin nhAt - duge phét biéu dau tien bai G.Lejeune Dirichlet (1805-1859), mot nha ton hoc Die g6c Phép, nhu sau: "Néu nhét n+ 1 con thé vao n cAi chudng (n € N*) thi ludn o6 (ft nhét 14) hai con thé bi nhét trong cing mot chudng". Mot c&ch téng quét, ta c6 nguyén ly Dirichlet mé rong: "Néu nhét m con thd vao n céi chuéng (m,n € N*) thi luén tén tai mot chudng chita ft nbd a1 + [ ‘m—1 con thé", 6 day, ky higu [a] duoc ding dé chi phan nguyén cia s6 thuc a tite 18 s6 nguyén lon nh&t khong vugt qué a. Ding phung phép phan chting, ta c6 thé dita ra mot céch ching minh khé ngéin gon cho nguyén ly Dirichlet (ngay c& dudi dang mé rong); hoc sinh THPT cing c6 thé lam duge vige ndy; va diéu dé khng hé lam gidm di gié tri cia ban than nguyén ly. Nguyén ly Dirichlet ¢6 rét nhiéu img dung (higu qua dén bit ngd): si dung n6, ta c6 thé ching minh duge nhiéu két qua séu sic cia toin hoc. Chinh vi vay, tai cdc cude thi hoc sinh gidi ton (quéc gia va quéc té), nguyén ly Dirichlet thutng xuyén dude khai théc. Dé minh hoa, duéi day, ta xét mét s6 bai toén cu thé. Bai 1 (Putnam 1993). Cho mét day s6 gdm 19 86 nguyén duong khéng vust qué 93 vd mét diy s6 gm 98 86 nguyén duong khong vugt qué 19. Ching minh ring tit hai day sé dé ta cé thé lan lugt trich ra hai day con cé téng cdc sé hang la bing nhaw. Giai. Ta xét bai toan téng quat: Cho hai day (hitu han) cdc s6 nguyén dung: BM StQS--StmSn, WS ys Sym Sm. Khi dé, tén tai céc "chi s6" 1 < iy < ip Sm, 1S i < jo 1 va) MS bytw—1 + Uy) — Op = OS M— Use S byipy-1 — ap Fy < bya, méu thudn v6i dinh nghia cia f(p). Mau thudn dé chiing té ring: qua thuc, mdi higu (1) 18 bé hon m, Bay gid, néu mot trong cdc higu do trigt tiéu: byip) — ap = 0, thi r6 rang ta c6 ngay dpom véi céch chon i := 1 =: fi, ia = p, jn := f(p). Trong trutng hop cdn lai, theo nhan xét trén, toan b6 m higu 6 (1) déu thude ZN [1,m — 1}, mot tap hgp chi 6 m —1 phan tit, nén theo nguyén ly Dirichlet, c6 hai higu bling nhau; tite 1A, tén tai r,8€ZN{l,m], r>s, dé Byer) — ar = by(s) = Oy => byte — bya) = Oy — O53 Va ta cling 06 dpem, v6i i = 8 +1, i= 1, f= f(s) +1, j= f(r). Bai 2 (Vo dich Cong hoa Czech 1998). Cho X la mét tap hop gém 14 s6 nguyén dwong phan bigt. Chting minh ring cé mot 56 nguyén duong k <7 va c6 hai tap con k-phin tt: {ar} a25...; a4}, {b159;..-3 be} ri nhau ctia X sao cho (aade _ a 1) a ay” ap al Gm +S iar Gidi. Xét Cf, = 3432 tap con 7-phan tit cia X. Téng (céc) nghich do cita céc phan tit 11 1 trong méi tép con nay 16 rang I8 khong vugt qué ++ 5++---+ 7 < 2,6 nén phai thude vio mot trong sé 2600 nita khong: 0 1 1 2 (saan' sana): (eon! soa 38 Theo nguyén lf Dirichlet, tdn tai hai tap con khée nhau c6 tng nghich do cée phin tir thude vao cling mot nita kho&ng. Logi bé khéi hai tap con dé oéc phan tit chung (hai tap con 7-phan tit khéc nhat thi c6 t6i da séu phan tit chung), ta sé thu duge hai tap . con k-phin tit (véi k nguyén duong, k < 7), thod yéu edu cita bai toan: higu cia hai téng nghich dio céc phan tit trong hai tap con nay sé sai khéc nhau ft hon 1/1000, Bai 3 (Iberoamerica 1998). Cac dai dign ctia n quéc gia (n € N*\{1}) ngdi quanh mét ban trén theo cich sao cho hai ngudi ngdi sdt bén phdi hai dai dién bat kj véi cing quédc tich thi phdi c6 quéc tich khdc nhau. Hay xdc dink (theo n) sé lin nhAt cé thé 06 cdc dai dign ngéi quanh ban tron dé. Gidi. Ta sé ding X,, X2,...,Xq dé ky higu n quéc gia c6 dei dign ngdi tai ban tron; céc dai dién 06 quéc tich X, (N* 3 i 2, j < 1999. ‘Tw céch xay dung trén, dé dang ching mink A(m; j) < A(n; J), A(m;j)/A(n; 3) vi moi bé ba s6 nguyén ducng m,n, j ma j < 1999 vam < n. Tir céch xay dung trén, cing d& thay (vdi mdi i € Nt AGi1), AG 2)... A(G 1999) l& 1999 s6 nguyen duong lién tiép (khong bé hon a1) do 6, theo gid thiét ciia bai tosn vé tap hyp A, thi tin tai j, € ZM[1; 1999] d8 A(i;j,) € A. BAy gid, Vi ji, ja... 52000 € ZN(L: 1999], nén theo nguyén ly Dirichlet, c6 hai s6 nguyen dung m 7 x 256 nén theo nguyén If Dirichlet ¢6 tam hoc sinh (khéc nhau) Aj, Aa,--+Ag, ma bai lim ciia ho thi tmg véi cée bd thudc ching:mét tap con A ndo 6 (trong s6 256 tap con néi trén). Nhung 2000 ~ 8 = 1992 > 7 x 256, nén lai theo nguyén lf Dirichlet - tdn tai tam hoc sinh (khéc hau, trong s6 1992 hoc sinh cdn lei), 1a Bi, Ba, ..., Bs, 06 bai lim ting véi céc bd thude cing mot tap con B nado dé (trong s6 256 tap con néi trén). Cudi cing, vin c6 1992 ~ 8 = 1984 > 7 x 256 nén ding nguyen ly Dirichlet mot lén nifa, tit 1984 hoe sinh cdn lai, ta tiép tuc tim ra tam hoc sinh (khde nhau), I C,C2,...,C, ma bai lam cia ho img véi cée bo trong cing mot tap con C ndo d6. Tit bén hoc sinh bat ky trong s6 24 hoc sinh Aj, Aa, +, As; Bi, Ba, ..-, Bs, C1, Cay... Ca, ta Iuén chon ra dutge hai hoc sinh 6 bai lam tig véi céc bd thudc cing mot tap con (mét trong ba tap con A,B ho§c C); gid sit, ching han, dé JA céc hoc sinh A; vA Ay (i,j € Z; 1 8 x 3 theo nguyén ly Dirichlet, ett 25 hoc sinh thi tim duge bén hoc sinh (trong sé 25 hoc sinh nay) cé bai lam tig véi bén bd Z,y,z,t € DC S doi mot khée nhau; va theo (3), hai hoc sinh ndo trong s6 bén hoc sinh dé cing c6 bai Tam khéc nhau 6 ft nhdt 18 hai cu héi, ding nhu yeu cdu cia bai tosn. ‘Tit 1/ va 2/, ta théy: s6 ty nhién bé nhAt phai tim 1k n = 25. 41 Bai 6 (Dé nghs, Todn 11, kj thi Olympic 30/4 nam 2006 chiing t6i dé dua theo ¥ cia mét dé thi Vo dich Romania 1997). Goi A la tap hop tt cd céc 66 bax = (a1, 22,3) ma 22,3 € (0, 7]NZ. BO x = (21;29\23) € A duge goi ld troi han 66 y = (is yaiys) € A, néu x Ay vd 2; > y; vdi moi i € {1;2;3}; khi d6, ta viet x > y. Tim sé ty nhien n bé nhit sao cho moi tap con n-phén tit cia A déu chia ét nhét la hai b6 jy maar y, Giai, 1/ Truéc hét, xét tap hgp B= {x € Alzy + 22+ 23 = 11}. Cé thé kiém tra truc tiép ring B 1A mdt tap con 48-phan tit cia A (B gdm ding: 4 phan ttt cé dang 2 = (0; 29; 11—22) void < 22 <7; 5 phan tit cé dang 2 = (1} 22; 10-29) voi3 < 22 <7; 6 phn tik c6 dang x = (2; 22; 9-2) vi 2 < x <7; 7 phan tit c6 dang x = (3, 22,8—z2) v6i 1 < 22 < 7; 8 phan tit c6 dang (4,%2,7 ~ a2) véi 0 < 22 < 7; 7 phan tit 6 dang 2) V6i 0 S 2 < 6; 6 phan tit c6 dang x = (6;2;4 — 2) voi 0 y. 2/ Tiép theo, cho N 3 n > 49, va B la mot tap con n-phan tit bat ky clia A. Ta sé ching minh ring B chtta it nhAt hai bo z,y max > y. Muén vay, xét céc tap con sau day cita A := {ee Alny 2 € Al(ey Vite = T},Co i= {2 € Al(a) =1V 22 < 6) V (21 > 1 Az, =6)}, 2Va2S5)V (a, >2Am=5)}, Coss {2 © Al(z = 3022 <4) V (01 2322 =4)},C Ae. UC = {a € Alm > 4V 22 <3} {2 € Clas = J} (i,j EZ 1 17 phan td; nhung chi c6 16 tap con Dp, (p,q € Zi4 << ps7; 0 y (dpem). Tit 1/ va 2/, ta thay: s6 ty nhién bé nhét can tim IA n = 49. 42 Mot sé phuwong phap giai cdc bai toan té hop nang cao Dang Hing Thing ‘Trong khoa hoc cing nhu trong di séng ching ta thutng gap bai ton xdc dinh s6 luong cée d6i tuong cé mot tinh cht ndo d6. Ta goi dé Ia bai toén dém.Té hop 1 mot nganh ton hoc nghién cttu céc bai ton mang cfu tric rdi rec trong dé cé bai toan dém. Kj nang va kién thitc ciia toén té hop IA rat can thiét cho nhiéu khoa hoc tit kinh té t6i sinh vat, tin hoc , h6a hoc va quan trj kinh doanh. Nh@ céc phuong phép ma té hgp cung cp, ching ta c6 thé xdc dinh duge s6 long céc phan tit cia mot tap hgp mét céch nhanh chéng va chinh xée ma khong cdn ( vi nhiéu khi cling khong thé) liét ké duge vi 86 d6 rt lén. Chuuong trinh phd thong (Iép 11) da trang bi cho hoc sinh hai quy t&c d&m ea ban(quy tée cOng va quy tie nhén) , céc khéi niém hoén vj , chinh hop, td hop. Nhd dé hoc sinh c6 thé gidi dude cée bai toén té hop co ban, tutong d6i don gidén. Tuy nhién d6i vdi nhiing bai todn té hgp phic tap, cn cé nhiing céc phyiong php "cao cip" hon,skc bén hon, Bai gidng nay nhim cung c&p mot 6 phuong phép dé 06 thé cong phé duge cde dang ton khé trong té hap. 1 Quy t&c cng téng quat Ban chét todn hoc cit quy t&e cOng( phat biéu cho cong viée voi nhiéu phitong én) 1& cong thiic tinh s6 phan tir cia hgp n tap hgp hitu han doi mot khong giao nhau. Cy thé ta 06 Cho n tap hop Ar,...,An ddi mbt khong giao nhau. Khi dé JA1UA2U...An| = > 14s Trong mhiéu bai toan té hyp, ching ta phai tinh s6 phan tit cia hop n tap hop bat ky ( khong nhét thiét rdi nhau). Khi d6 ta c6 quy tic cong cho s6 phin tit cia hop clan tap hop bat kj , thutng duge goi IA cong thtéc bao hm va loai tri. Dinh ly (Cong thtte bao ham va loai trit) 43 Cho n > 2 tap hgp hitu han Aj,..., A, . Khi d6 ta cd [Ay U Ag U..Anl = =SlaI- Slain Ad+ SO [Ain ayn al idan 1 iGjek on feet SY ya, 944. Agl 1 ackencty n ek (1A Ag... An Dinh ly nay c6 thé chimg minh tuong déi d& bing phuong phép quy nap , xin dinh cho ban doc. ‘Vi du 1 Trong tap S = {1,2,..., 280} c6 bao nhiéu s6 khéng chia hét cho 2, 3, 5,72 Cidi ‘Ta dém xem trong tép S c6 bao nhiéu s6 chia hét cho ft nh&t mot trong cée s6 2, 3, 5,7 Kf higu Ay = {k € 3: k chia hét cho 2}, Az = {k € S: k chia bét cho 3} Ay = {ke S :k chia hét cho 5}, Ag = {k € 5: k chia hét cho 7}. Khi d6 Ay U Ag U AgU Ag IB tap cée s6 chia hét cho it nh&t mot trong cdc s6 2, 3, 5,7 Ta c6 2 2 [Ail = 282 = 140; a] = (282) = 99; 4s] = 222 = 56; 144] = 28° = 40 2 3 5 7 280 80 [Ai dal = FP = 465/44 As = 2 = 28;|A,N Al = [2M Aal = PRP = 385140 dl = (222) = 13514979 Aad 15 21 280 [Av Aa Adl = [P| = 95141042 0.Adl = 280 |ALN Ag M1 Aal = Ug) = 451420 Asn Aad [AL Ag Ag 9 Aah = [222 2 ‘Sit dung cng thite bao him va loai trir ta thm duge |Ay U Ag U Ag U Ay| = 216. Thanh thi, trong tap S c6 280 — 216 = 64 s6 khéng chia hét cho 2, 3, 5, 7. ‘Vi du 2 (Cong thitc ham Euler) Voi méi sé nguyén duong n > 1, ky higu 9(n) Ia s6 cde s6 nguyén duong bé hon n va nguyén t6 véi n. Ham ¢(n) duge goi la him Euler. Né dong mot vai trd quan trong trong nhiéu bai toan s6 hoc. Ching minh ring trong d6 71, P2, ...,Pm 1a tht cd céc ude nguyén té phan biét cia n, 44 pape cee Gadi Kj higu $ = {1,2,....n}. Ta dém xem trong tp S'c6 bao nbiéu s6 chia hét cho it ah&t mot trong céc 86 p1,....Pm Goi Ai = {k € S: k chia hét cho pi} (i = 1,2,...m). Khi dé Ai U Ap... Am la tap céc 86 chia hét cho it nh&t mot trong céc $6 pi, ..sPm- Ta cé n Pi:Pig--Piy 145, 14g. Aal Do dé theo dinh ly 1 JA, U.A2 U... Amel = n n Pi i & n PsP n fee YS pt Lists PuPa~Pis geek (® PLP Pm Vi @(n) chinh bing s6 cfc s6 khong chia hét cho tht c& cfc $6 pi,...Pm nén n— [AU A2U... Am = 2 Thiét ké cdc céng doan thich hgp Dé &p dung duge quy tii han didu cét yéu IA phai thiét ké mot mo hinh gdm viée thuc hign lign tiép cdc cong doan. Quy tée nhan phét biéu: V6i mi céch thyc hién 3 cong doan trudc thi cong doan thit k oé thé lam theo n, céch. Nhu vay: s6 céch thue hién 6 méi cOng doan phai khGng phy thudc vao cach ndo da duge thuc hién 4 cong doan truée d6.Thanh thit, muén sit dung duge quy téc nhn , trong mé hinh cia ta gém viée thyc hién lign tiép c&c cOng doan, 86 céch thyc hién & méi cOng doan I& phat nhu nhau véi moi céch da dge thyc hign 6 cOng doan tribe 46 Vi du 3 C6 4 ngudi A,B, C, D can chon vao chite giém déc, ké toan trudng va chi tich HDQT.Gié sit vige chon nhén sy phai théa man yéu cdu : Ong A khong thé duge chon lam giém déc, chite chit tich HDQT phai la ng C hoe Ong D. Héi cé bao nhieu ech chon, 45 Giai: C6 mét Idi gidi nhu sau: Viée chon ba vi trf giém déc, ké todn trudng va chi tich HDQT tién han theo 3 cong doan: Cong doan 1: Chon giém d6c: C6 3 cach chon chite gidm déc ( chon B,C,D). Cong doan 2: Chon K6 Tofn truéng : Cé ba céich chon ké toén trudng tit ba ngudi edn lai . Cong doan 3: Chon Chi tich HDQT C6 hai céch chon ( 6ng C hoe ong D) Theo quy téic nhan thi s6 céch A 3.3.2=18. Och nay khong déng: Vi s6 céch thyc hign cong doan 3 phy thudc vao két qué cia céc cong doan 1,2 trude dé : Néu 8 céc cong doan trudc , 6ng C va ong D khong dite chon thi cong doan 3 méi cé hai céch. Can néu C ho&e D da dugc chon thi 8 céng doan 3 chi c6 mét céch hose tham chi khong c6, ‘Tuy nhién néu ta thiét ké vie chon ba vi trf gidm déc, ké todn trudng va chi tich HDQT tién hanh theo 3 cong doan khéc thi vin c6 thé ép dung quy tée nhan, Cy thé Cong doan 1: Chon Chi tich HDQT: Luon c6 hai céch chon: C hog D. Gong doan 2: Chon Gidm déc: Ta Iudn o6 06 hai céch chon da é két qua cia cong doan 1 thé no Sau cong doan 1 cdn ba ngudi trong dé o6 ong A. Bé ong A ra ta cdn hai ngudi c6 thé chon vao chite Giém déc. Cong doan 3: Chon Ké Toén truéng : luon 6 hei céch ( tit hai ngudi cdn Iai ) Vay két qua 18 06 2.2.2=8 céch chon. Day Ia dép s6 ding, ‘Vi du 4 a) Gid six c6 8 van dong vien béng ban tham du mét gidi du. Trong vong dau cia gidi, ban t6 chic edn phan ra 4 cap du. Héi o6 bao mhiéu céch ghép thinh 4 cp diu? b) Gia sit c6 2n van dong vien béng ban tham dy mot gidi déu. Trong vong diu cia gidi, ban té chite cin phan ra n cp déu. Hdi ¢6 bao nhiéu céch ghép thinh n cép dau? ©) Tit b) chiing t8 ring v6i mdi n € N* ta c6 (n+1)(n-+2)(2n — 1)(2n) chia hét cho m. Giéi a) Ta thiét ké vige thyc hin chon theo cée céng doan sau: Cong doan 1: Chon 2 ngudi trong 8 ngudi lam thin c&p du thit nhit. CO CG} céch chon. Cong doan 2: Chon 2 ngudi trong 6 ngudi cdn lai dé lam thanh c&p du thit hai. C6 C8 c&ch chon. Cong doan 3: Chon 2 ngutai trong 4 ngudi cdn lai dé lam thanh c&p du thit ba. C6 C? céch chon. Céng dogn 4: C6 3 = 1 céch chon: Hai ngudi cdn lai sé lam thanh c§p déu thit tu. Theo quy tc nhan c6 C3.C3.C2C} céch chon. Vi thit ty 4 c&p du khong duge xét dén nén s6 céch ghép thinh 4 cp dau lA C3.03.03.03 = 105. 46 b) Ly luan tumg ty nhu trén, s6 céch ghép thanh n cap déu la _ hy Cyn C203 _ nl c) D8 bién adi (2n)! _ (n+1)(n +2)(2n — 1)(2n) nla * ViT I mot s6 nguyén dung nén cong thite nay chting t6 (n +1)(n+2)(2n — 1)(2n) chia hét cho 2” 3 Sit dung phép song 4nh C6 n ngwdi dén dy mét budi néi chuyén trong mot hoi tru’ng gém 200 ghé. Gid sit moi ngudi chi chiém mot ghé ngdi va méi ghé chi cé nhiéu nhét mot ngudi ngdi. Néu ta duge thong béo ring moi ngudi déu cé ché ngdi thi ta két luan duge n < 200 Néu ta biét them ring khong c6 ghé ndo tréng thi ta biét ngay JA n = 200. Néu c6 mot s6 ngudi phai dmg vi khong c6 ghé thi ta suy ram > 200. Nhuu vay cé thé xéc dinh hay uéc luong s6 phan tit cia mét tap hgp A nado dé thong qua mot tap hop B ma ta da biét 86 phan tit cita B nh’ mot phép tung ting (nh xa) gidia A voi B.. Day 14 mot phitong phap dude sit dung rat hiéu qua dé gidi nhiéu bai ton dém nang cao. Cho nh xa f: A> B, « Anh xa f duge goi 1a mét don 4nh néu véi hai phn tit bit ky a1,a2 € A, néu a; # ay thi f(a1) # f(aa). Tite Af(a1) = f(a2) + a1 = a2. * Anh xa f duge goi 1A mot ton énh néu vdi moi b € B déu tén tai a € A dé F(a) * Anh xa f duge goi lA mét song anh néu moi b € B, tén tai va duy nht a € A dé f(a) = b. Noi céch khéc f 14 song Anh khi va chi khi n6 déng thdi 18 don énh va toan énh . Dinh ly Cho A va B a hai tap hgp hitu hen. + Néu od mot don énh f : A B thi JA] < |B © Néu c6 mét toan anh f: A — B thi |A| > |B| * Néu c6 mot song énh f : A B thi [A] = [BI Vi dy 5 Cho tap A = {1,2,...,2n}. Mot tap con B cha A goi la mot tap can néu trong tap dé sé céc 86 chin va s6 cdc s6 lé bing nhau.( Tap réng 1A mdt tap cn vi sé. céc 86 chdn va s6 céc s6 1é trong tap rong déu bing 0). Héi cé A 6 chita bao nhiéu tap can? Ching han v6i n = 2,A = {1,2,3,4} . A c6 6 tap con cn Ia céc tap sau 0, (1, 2}, (1, 4}, {2, 3}, {3, 4}, (1,2, 3,4}. a7 Gidi: KY higu X = {2,4,...,2n} la tap hop tht od céc s6 chin cia A va Y = {1,3,...,2n—1} IB tap hop tht cd céc 36 18 cia A. Goi C 1a ho t&t cd ede tap can cha Ava D 18 ho céc tap con cite A cé ding n phan ti. Ta lap mot énh xa f titC vao D nhw sau: Gid sit B 1A mot tap cin . Ky higu By, By tuong ting la tap cée s6 chin vA tap céc $6 18 cia B. Khi d6 dat F(B) = BOY \ By). Do B la tap can nén|B;| = |Ba|.Thanh thit | f(B)| = |By|+|¥ \ Bal = [Bil +[¥|— 1Ba| = I¥| =n. Vay f(B) €D, Tiép theo ta ching minh f 14 mot song énh. + f la don énh: Gia sit f(B) = f(C). Suy ra By U(Y \ Ba) = Gn (¥\ G). Vi Bi, Cy la tap céc 86 chin ; (Y \ Ba), (Y \ C2) 1A tap eée s6 18 nén tit 46 suy ra By = C1; (¥ \ Ba) = (Y \ C2) . Do d6 B, = Cy; By =O + B=C.. + fT mot toan anh: Gid sit M € D la mot tap con cia A c6 n phan tit. . Ky hieu M1, Ma tuong ting la tap céc s6 chin va tap cfc s6 18 ctia M. Dat By = My; By = ¥\ Mp va B= B\U By. Ta cé 1Bal = [Mas |Bal = ¥| — [Ma] =n ~ [2M] = [MI ~ [Mal = [Ma Vay |Bil = |Bal do d6 B 1a mot tap can, Ré rang f(B) = ByA(¥\ Bs) = MyUM; = M. Vi c6 mot song anh gitta ho cfc tap cén va ho céc tap con c6 n phan tit cia A nén theo dinh If tren s6 cfc tap can cia A bing s6 cée tap con o6n phan tit cla A. Vaya 6 tht of 18 CR, tap can, Vi du 6. Cho trudc s6 nguyén duong n va s6 nguyén duong r tho& min r 1 ( do aig1—a; > 2). Dodd by < dy << Snr $l Ta dinh nghia f(A) = B Ta o6 B € B, do vay f 14 mot énh xa tit A vao B. Ta s& ching minh f 1a song anh. + f Ia don énh: Ty cong thite by = a; ~(i- 1) = a; i +1 suy rag; =b +i Do d6 néu f(A) = f(A’) thi A= A’ + £18 todn anh: Gid st B= {by < be < ++ 11010111 (1,4,1) — 10111101 (0,2,4) > o1101111 (3,0,3) — 11100111 Ro rang phép tung ting d6 18 mot don Snh. Nguge lai, voi mdi day 8 ky ty v6i 6 ky tu 1 va2 ky tu 0 khi ta dém tir tréi sang phai m& c6 :a s6 1 lien tiép ,s6 0, 0 86 1 lien tiép , s6 0 vac chit s6 1 lien tiép thi day dé sé ting voi bo (a, b,c) thod min a+b-+c= 6. Chang han day 10110111 sé ting véi bd (1,2,3) tite 18 ting voi sy Iya chon : 1 kem xodi,2 kem 50 cdla va 3 kem sita, Day 01011111 ting véi bo (0,1,5) tite 18 tng voi sy Iya chon 1 kem 56 cdla va. 5 kem sifa, Nhu vay ta da thiét lap mot song anh gitfa tap hop céc su Ifa chon véi tap hop cée day nhi phan d6 dai 8 trong d6 06 6 ky tu 1 vA2 ky tut 0. Do d6 86 eée suf Iya chon bing 86 céc day nhi phan d6 dai 8 trong d6 06 6 ky ty 1 va 2 ky ty 0. Mat khée mot day nhi phan d6 dai 8 voi 6 ky tu 1 va 2 ky tu 0 tuong ting véi céch chon 2 vi tri trong 8 vi tri dé ghi s6 0 ( 6 vi trf cdn lai ghi s6 1). Thanh thit c6 C? = 28 day nbj phan do dai 8 véi 6 ky tu 1 va 2 ky tut 0. Do dé s6 cde su lta chon 18 28. b)Mot sv Iya chon : " a kem xoai, b kem s6 c6 la va ckem sita " dugc ky higu béi mot b@ ba (a,b, c) trong dé a,b, c IA cde s6 nguyén duong thod man diéu kigna+b+c=6. 49 Voi m8i b9 (a,b, ) thod man diéu kign trén ta cho tuong ting véi b9 (2, y, 2) véi Premed tea Garena Ca ee} nguyén khong am thoi man ditu kien +y+2=a46+c-3=3, D8 kiém tre Hing day 1A mot phép song énh Billa tap céc b6 ba (a,b,c) trong dé a,b,c 1A céc s6 nguyén duong thod man diéu kin a+b+e=6. véi tap céc bo ba (2, y, 2) 1d cfc 86 nguyén khong am thod mn digu kién T+y+z=at+bt+c-3=3. B&ng suy luin tutong ty nhu cau a) ta tim duge s6 cdc su Twa chon 18 G2 = 10. Chit yj: Ta c6 bai todn téng quét sau: Mot cia hing kem 6 bén m loai kem . Mot nhém C6 nm ngudi vio an kem va goin c6c kem. Héi 8) Ho ¢6 tét cA bao nhieu su Iya chon? 5) Ho 6 tht 8 beo nhieu sy Ita chon trong &6 cdi m loni kem déu 6 mat? Dép sé: a)Cr1 BORG 4° Dang cong cu sé phic Vi du 8 Cho p lA mot sé nguyen t6 va m € N*. Gia sit po B= {(h1y bp) 0S m1: Dik =0 (mod Dp} a Xe dinh s6 phan tir cia tap [| Giéi V6i méi r = 0,1,...,p— 1 ta dink ngbia tap E, E, = {(h, P(e) = ED a*) = = ae ti ok mt Gia sit F(z) = D® ane” . Dat pa An = {(hy tps) iki =n} an = [Anl 50 Khi d6 E, = |E, Unssr (mod pn, Y l= Yo a. ner "(mod p) ner (mod p) Goi w 1A mot nghiém phitc tuy § cha o(2) = 2 +... +241=0.Tacdwi #1 WIS j wi" = wu. Viho {jm}, j = 1,2,..,p—1 a he thing du thu gon (modulo p) nen{w",i = 1,2,..,p— 1} = {w,j 1,2, ..p~ 1}. Suy ra F(w) = 1. Dat b = |B]. Vin =r (mod p) + w" =w" nén tit (1 ) suy ra = owe = Per @) 1= Fi = = Dat Q(z) = TPT} bea + bo ~ 1. Theo (2) voi mi nghiém w of g(z) ta c6 Q(w) = 0. Do 6 tdn tai hing s6 @ Q(x) = ag(x) + bya = +++ =b; = by — 1. Tadd Hence Trudng hop 2 plm. Khi dé wim 0> FW) =0+0= Fl) = Mg ane = DPT bud” 9 Opa C6 «Do vay v6i mi r = 0,1,2,...,p—1 ta IE) = ‘Tom lai Vi du 9 Cho p > 2 1a 6 nguyén t6 1é va 86 nguyén duong n > p. Xéo dinh s6 cic tap con A cia $= {1,2,...,.n} thod man hai digu kign sau i) |Al=p ii) S(A) = 0 (mod p) trong 46 S(A) 1a téng céc s6 nim trong A. Gidi V6i mdi r = 0,1, 2,....p— 1 goi my la 86 céc tp con A cia S thda man r (mod p) = Diet. Cé dinh mot nghigm phite bat ky w cia f(z). Ta 06 {u', 1,2,...,p~ 1} la p— 1 nghiém phan bigt ota f(x). (That vay nhyt trong vf du 8 da chi rau! £1, (WP = (uP)! = 1. Néuut ud ud = 1 pli — 7 mau thulin ) Do do p ~No(2) = [[(e- Xét da thite Q(2) = [T.y(x - w*) = D2, aia*. Theo dinh ly (3) > www? = (1) an p 1 p thi ng cia vi du trén voi n = 2p. QR=k (modp), 646k = [2] 5 Thiét lap quan hé truy héi ‘Vi dy 9 (Bai todn thép Ha noi) Tuyong truyén ring tai mot ngoi thap & HA noi cé mot tam dé bing ding trén d6 c6 dat ba chiée coc bing kim cithg.Litc khai thién lap dia, trén coc s6 1, Phat t8 Nhu Lai da xép 64 chiée dia bing ving 6 dwéng kinh khéc nhau sa0 cho cc dia 06 dutng kinh Ién hon xép & dui, céc dia 6 phia trén cing 6 trén cao cang nhé dan. Cac nha sur dude yéu cdu chuyén tt cd céc chiée dia d coc 56 1 sang coc 86 2 v6i quy t&c eau: - Méi lan chi duge chuyén di mot chiéc dia. ~ Trong qué trink di chuyén khong duge dit dia Ién lén trén dia nhé ( do dé cdn thiét phi c6 thém chiéc coc trung gian thit ba). Gid sit méi lan chuyén mot chiéc dia mt mot gidy. Héi céc nha su cn ft nhAt 1a bao nhiéu nim dé chuyén tit cA céc chiée dia & coc sé 1 sang coc 86 2? Giti Gia sit lie dau trén coc 86 1 06 m chiée did. Goi up 18 s6 In it nh&t dé chuyén tht cf cée chiée dia & coc 86 1 sang coc s6 2. Ta thit tinh mot vai gid tri cia uy Vin = 2. Ta cn thyc hién ba phép chuyén sau -Chuyén dia bé sang coc s6 3 - Chuyén dia lén sang coc s6 2. ~ Chuyén dia bé va coc 6 2 Vay = 3 Voi n = 3. Ta can thuc hign theo ba giai doan sau - Chuyén hai dia 6 phfa trén sang coc s6 3. Nhu da théy 6 trutmg hyp n= 2, ta cn 3 phép chuyén. 53 - Chuyén dia 1én nhAt sang coc s6 2 - Chuyén hai dia 4 coc sé 3 vé coc s6 2. Nhu da théy 6 trutmg hop n = 2 ta cin 3 phép chuyén. Vay ta cin 3+1+3=7 phép chuyén. Do d6 ug = 7. ‘Trudng hop n = 3 ggi ¥ cho ta thiét lap he thite truy héi ma day (up) phai thod man. Dé chuyén duge n chiéc dia theo quy tc trén ta phai thyc hién theo ba céng doan sau: - Cong doan 1: Chuyén (n — 1) dia 6 phia trén chiéc dia lon nhAt sang coc sé 3 theo quy téc trén. Ta can un; phép chuyén. Chiéc dia lén nhAt vin gitt nguyén 6 coc s6 1 Khi di chuyén (n — 1) chiée dia 6 tren né. ~ Cong dogn 2:Chuyén dia lén nhAt sang coc s6 2. - Cong doan 3: Chuyén (n — 1) dia coc 86 3 vé coc 56 2 va dat len trén chiée dia 16n nh&t Ta céin un; phép chuyén. Do vay dé chuyén n chiéc dia tix coc s6 1 sang coc 86 2 ta cin un + 1+ Unt = 2tm-1 +1. Vay ta c6 hé thite truy hdi sau Un = Qn + 1, Tut he thitc truy hdi nay ta cé thé lap duge cong thite cia s6 hang téng quét cia day. Bling quy nap dé chimg minh duoc Un = 2 = ‘V6i n = 64 thi ug, = 18.446.744.073.709.531615, D6 lt s6 lan chuyén dia mA céc nha sut phai thuc hign dé hoan thanh cong vige. Néu méi lan chuyén mot dia mét 1 giay thi cdn dén hon 500 t} nim céc nha sit méi chuyén duge tit c& 64 chiéc dia sang coc 36 2. ‘Vi du 10 (IMO 1979, bai s6 6) . Gia ott A va £ la hai dinh déi dign cia mot bét gide déu. Mot con éch bat dau nhdy titdinh A. Tai mdi dinh cita bat giée (trit dinh B) , m6i ct nhiy con éch chi c6 thé nhay t6i hai dinh ké véi dinh 6. Khi con éh nhay vdo din E né sé bi ket vinh vién 4 dé. Cho trude s6 nguyen duong n . Héi véi n oft nhdy, 06 bao nhiéu cach dé con éch nhéy vao dinh B. Gidi . Goi ay 1886 cach dé con éch nhay vao dinh B. Dé thfy a; = a2 = a3 = 0; a4 = 2 Gia st tir A theo chigu kim déng hé cdc dinh lan lugt ls A> BC D+ EB — FG — H — A. Tit A con éch dén B phai qua mot s6 Ié bu6c;Ttr B con éch dén C phai qua m@t s6 18 bude.Tit C con éch dén D phai qua mot 6 18 buée.Tit D con éch dén E phai qua mét 86 lé buéc. Vay s6 buée dén E ditt khost phai 1A mot s6 chin, Néi cach khée néu n 18 thi kh6ng ¢6 céch ndo nhay vao B. Vay azx-1 = 0. Ta cn tinh any, k > 1 Xuét phat tir A,véi hai buée nhdy dau tién con éch 06 thé 06 cée céch sau NASBA NQAGHoA 2QAWBAC )ASHAG Néu theo céch 1) thi s6 céch tdi E 1a az4-2.Nén theo céch 2) thi s6 céch téi E 1a 024-2. Goi Cn, gn ln lut 18 s6 cach dé con ch, xudt phat tuttong ting tir C,G, nhay vao dinh E voi n ci nhay. Vi ly do déi xing ta c6 cy = ga. Vay néu theo céch 3) thi 6 céch. tdi E 1A c2p-2; néu theo cach 4) thi s6 céch tdi E 1A goe-2 . Theo quy tc cong ta c6 Ang = Og4—2 + Oae—2 + Cok-2 + Gok-2 = 22-2 + 2eyp-2 (5) Xuét phét tir C,véi hai buée nhdy dau tién con éch 06 thé c6 céc cAch sau : 1)O4+BOA %wW)C4+B4C 3) C9 DC 4c) C4 D> EB Néu theo céch 1c) thi s6 céch téi E Ia az4-2,Néu theo céch 2c) thi s6 céch t6i 1a cax-2.Néu theo céch 3c) thi s6 céch ti E 18 c7,~2.Néu theo céch 4c) thi s6 céch t6i E 1a 0. Theo quy téc cong ta c6 Cae = 24-2 + 2erk-2 6) ‘Tix (5) va (6) riit ra cm, = age — O24-2 > Crk-2 = 24-2 — 24-4. Thay vio (5) ta duge nx = 4a2k-2 ~ 2are—4 Dt uy = are ta 06 Up = Attn ~ 2u_-2) tH = Oe = 05 Ue = Oy = trinh dic trimg ta di dén cong thiic sau (2+ v8) = (2 vay . Bling céch gidi phuong a te k= 1,2,.0 Vi du 11 Cho sé nguyén duong n va $ = {1,2,...,n}. Tim s6 céc tap con (ké cd tap r6ng) cia S m& khong chtia hai s6 nguyen duong lién tiép. Gidi Goi aq 1& 86 phai tim. D@ thy a; = 2,a = 3,a3 = 5. Ching han véin = 3.06 5 tap con thod 1a 0; {1}; {2}; {3}; {153}. Goi A, 1a ho cde tap con cé tinh chat da néu. M6i tap A © Ang2 gdm hai loai : Loai 1 gém céc tap chita'n + 2.Loai 2 gdm céc tap khong chifa n +2. Néu A la tap loai 1 thi A khong chifa n + 1. Do d6 néu bd di khdi A phin tirn +2 ta duge mot tap con cia A,. Ngugc lai véi mi tap con B cia Ay thi tap A= BU {n+ 2} 1a tap loai 1 cla Anse. Thanh thit s6 tap loai 1 18 a,. Moi tap loai 2 76 rang 14 mét tap con cila Any: vA ngugc lai. Thanh thit s6 tap loai 2 18 an41. Do dé ta cé quan hé sau Ong? = Ong +n Mat khéc véi day Fibonacy ta c6 Fas2 = Fai + Fn. Via = Fy = 2:09 = Fy = 3503 = Fy =5 tasuy ra a, = Fuso. Vay am? — pt? vB Oy = Fuga bd6a=48,a= eo ‘Vi dy 12 Cho s6 nguyen duong n va S = {1,2,....n}. Goi cy 1a 86 cdc tap con cia S ma chia ding hai sé nguyén dutong lign tiép . Chimg minh rng 2nFnoi — (n+ 1)Fa fa 55 Gidi Goi C, 18 ho ce tap con 6 tinh ch&t di néu. Méi tap C © Casa gdm ba loai : Losi 1 gém céc tap chita n+ 2,n +1. Loai 2 gm cée tap khong chitan +2. Loai 31a. céc tap chita n+ 2 nhutng khong chtta n+ 1. +) Néu C la tap loai 1 thi C cling khong chita n ( vi néu C chifan thi C chita 2 cp sé aguyén lien tiép 1A.n,n+1 vAn-+1,n+2) . Bé di khi C phan titn,n+1,n-+2 ta duge mot tap con cita {1,2,....m—1} khng chia hai s6 nguyen dung lién tiép, do dé n6 IA phan tit cia An1.Nguge lai véi méi tap con A cia A, thi tip C= AU{n+2,n+1} 18 tap loai 1 cita Cu42. Phép tuong tng ndy 1A song anh. Thanh thit s6 t€p loai 1 1a Qn = Fri +) M@i tap loai 2 15 rang I mét t&p con cia Caya VA ngudc lai. Thanh thit s6 tap Toai 2 18 cay. +) Néu C Ia tap loai 3 thi C khong chtta n+1. Do 46 néu bé di khéi C phan tin +2 ta duge mot tap con cita.Cy.Nguge Iai voi mbi tap con B cia C, thi tap C = BU(n+2} 1 tap loai 3 cia C,42. Thanh thi ta c6 he thite sau uta = ata + on + Fret Ti d6 bing quy nap, sit dung he.thitc truy hdi cia day Fibonaci ta c6 céng thie néu tren, £ te Xay dung song anh gidi mot sé bai todn td hop Huynh Tén Chau (THPT Chuyen Luong Vin Chénh, Phi Yen) ‘Trd ngai lén nh&t khi gidi mot bai toén té hgp 1A xée dinh huéng di. Ré rang dé c6 kha ning dinh huéng t6t thi vige rén luyén e&e phutong phép tiép cn Ia rét can thidt. Bai viét ndy gidi thigu véi cdc ben doc mét phutong phép hiéu qua trong nhiéu bai toan té hop ma ta tam goi lA phwong phdp song énh. Phuong phép nay dya trén két qué hién nhién sau: "Néu cé mot song dnh di tir mot tap hitu han X t6i mot tap hitu han Y tha luc luong (tic s6 phén tt) cia X va Y bing nhau" 1, Mot céch tu nhién, két qua trén hudng ching ta dén ting sit dung song énh dé so sinh luc lugag hai tp hop. Bai toan 1 (Vo dich Lién X0) C6 mot nhém ngudi ma trong dé, méi cap khong quen nhau c6 ding hai ngudi quen chung, cdn méi cip khong quen nhau thi khong cb ngwdi quen chung. Ching minh ring 36 ngudt quen ctia méi ngudi la nhw nhaw Ldi gidi. Gid sit a quen b va tap cée ngudi quen ciia a va b (khong ké a,b) 1A A va B Méi ngu’i a! thuge A sé quen véi duy nhét mot ngudi thude B (do a’ va b khong quen nhau, hon nia ho da cé mot ngudi quen chung JA a). Tuong ty, méi ngudi thude B cing quen véi duy nht mOt ngudi thuge A. Vay tdn tai mot song nh di tir A t6i B, tite a va b c6 86 ngwoi quen bing nhau. Néu a khong quen 6 thi tén tai c quen cA a va b, do dé s6 ngudi quen cia a va b bing nhau do cing bing sé ngudi quen ciia c. Bai toan 2 (Trung Quéc - 1997) Trong céc sau nhi phan cé do dain, goi am la sb cdc au khong chita qud 2 36 lien tiép 0,1,0 va by la s6 cdc cau khong chiia 4 sé lién tip 0,0,1,1 hodic 1,1,0,0. Chiing minh ring: buss = 2am. Léi gidi. Ta goi mot xfia thudc loai A néu n6 khong chita 3 sé lien tiép 0,1,0 va got mét xdu thudc loai B néu né khéng chita 4 s6 hang lién tiép 0,0, 1,1 hole 1,1, 0,0. V6i mdi xéu X = (21, 22)...,n), ta xy dug f(X) = (ys, yo,--.,Ynt1) abit sau: = 0, ve = 21 +22 +--+ + a4 (mod 2). R6 rang X chita 3 sé lien tiép 0, 1,0 khi va chi khi f(X) chita 4 sé hang lien tiép 0,0,1,1 hoi 1,1,0,0, titc IA X thuéc loai A khi va chi khi f(X) thuge B. 37 Vay f I mot song nh di tit tap céc xau loai A do dai n dén tap cdc xau loai B do dain +1 ma bit diu bing 0. Nhung tit mdi xau X thude B ta nhan duge mot xau X cling thuge B bang céch déi céc phan tit cia X theo quy té&e 1 — 0,0 + 1 nén s6 cée xau loai B c6 d6 dain +1 gp déi s6 céc xdu loai B dé dai n +1 ma bat dau bing sé 0. Ti 6 ta c6 dpem. 2. Tit vige so sénh Iyc lugng céc tép hgp, phuong phép song énh c6 thé gitip ching ta dém s6 phan tit ctia mot tap thong qua sit so sinh Ive lugng ciia tep dé véi mot tap khéc ma ta da biét s6 phan tit cia n6. Bai ton 3 (V6 dich Ucraina - 1996) Goi M la ctc s6 nguyén duong viét trong hé thap phan cé 2n chit sd, trong dé cé n chit sd 1 van chit sé 2. Goi N la sé tat cd cdc 86 viét trong hé thap phan cé n chit s6, trong dé chi cé chit s6 1,2,3,4 va sé chit s6 1 bang 56 chit s6 2. Chiing minh ring M = N. Lai gidi 1. Voi méi s6 06 n chit s6 gdm céc chit s6 1,2,3,4 va s6 chit s6 1 bing 86 chit s6 2, ta "nhan doi" thanh s6 c6 2n chit s6 theo quy t&c sau: dau tién, hai phién ban cia s6 ny duge viét ké nhau thanh s6 6 hai chit 56, sau d6 cfc chit s6 3 6 n chit s6 dau vA céc chit 56 4 4 n chit s6 sau duoc déi thanh chit s6 1, ce chit 86 3 6 n chit sé sau va cdc chit sé 4 6 n chit s6 du duge déi thanh chit s6 2. Vi du: 1234142 > 12341421234142 — 1212122121112, Nhut thé, ta thu duge mt s6 c6 ding n chit s6 1 van chit 96 2. Ré rang day 14 mot don énh. Dé chiing minh day 1A mot song énh, ta xy dig énh xa nguge nh sau: voi mbi 6 c6 n chit a6 1 vA n chit 96 2, ta o&t n chit s6 du van chit s6 cuéi rdi cong ching theo cot véi quy te: 1+1= 1, 2+2=2,14+2=3, 2+1=4, vata thu duoc mot 86 c6 n chit s6 gdm céc chit s6 1,2,3,4 vdi s6 chit 56 1 bing 86 céc 86 2. Vi du 1212122129112 — 1234142 nu sau: 1212122 1221112 234142 Nhu thé song énh gitta hai tap hop da duge thiét lap va ta c6 M = N. Céch xay dung song Anh nhv trén khé dep song hoi ciu ky. Ching toi da tim ra mot loi gidi ngén gon hon nhwt sau: Li gidi 2. Vk € S = {0,1,...,n}, ta thuc hign 2 lin (d9c lap véi nhau) vige dénh déu k trong n vi trf trén mot hang, Sau 46, 3 méi vi tri ta ghi: chit s6 3 néu duge dénh déu 3 lan, chit s6 4 néu khong duge dénh déu lan nao, chit s6 1 néu chi duge dénh du lin dau va chit s6 2 néu chi dude dénh déu lén sau. R6 rang Khi cho & chay trén S thi s6 t&t c& céc s6 thu duge chfnh 18 NV. Hon nita, Vk € S thi c6 thé xem s6 céch dénb du lan du 1a s6 céch chon k trong 86 n vi tri, cm 86 céch dénh déu lén sau 1a s6 céch chon n — k vi tri trong n vi tri. Do d6 tong s6 céch dénh du khi k chay trén S ding bing s6 céch chon n phan tit tit 2n phan tit (tic 1a C},), va s6 nay chinh la M. Vay ta c6 diéu phai ching minh. 58 Bai toan 4 Cho céc 36 nguyén duong n,k udin > k. Xét phép todn f déi véi bo sip thit tu X = (a1,...,aq) nhw sau: mdi lin chon k s6 lién tiép tiy 9 trong X va déi déu ctia chting. Tim 86 céc b6 thit ty X = (x,,...,tn) thda man cdc diéu kién: i) Moi phan tit ciia X déu thude tap {0,1}. ii) Co thé thuc hign hitu han lan phép todn f dé tir X mhan dugc b6 (1,2,-..,1) Ldi giai. Xét bo thit ty X = (x1,...,a,) tity Ta c6 2 nhan xét sau: 1) 06 ding n ~ k +1 nh6m k sé lien tiép. 2) Sau mét s6 chin lan thyc hign phép ton f cho mot nh6m k s6 lign tiép trong X thi gid tri & s6 dé khong déi. Nhu vay, m6i phuong én thyc hign hiu han lan phép ton f cho X tuong tg véi mét b6 nhj phan A = (a;,az,...,a,), trong 46 @, tinh theo modun 2 ciia s6 ln thyc hign f cho nh6m & 86 lién tiép (2,,2441,...,Tize-1), va X tr thanh (apace aaa CIEE MEG (AH). 3. Ciing tir vige so sénh luc lugng céc tap hop, phuong phép song anh 14 mot cong cu die luc dé thiét lap va ching minh céc cong thite t8 hop. Thong thuong, ngudi ta xay ding mét song 4th di tit mot tap vao chinh n6, vi nguyén tic 6 day c6 thé phat bigu nhu sau: "Khi dém s6 phan tit mot tap hgp bling nhiéu céch thi cée két qua thu duge bing nhau". Chang han tif bai ton 3, néu ta tinh N theo céch "truyén théng": c6 CC. céch chon vi tri cho i chit s6 1 vA i chit s6 2, con Iai 2"-* céich chon vi tri cho cée chit 56 3 va 4, véi i chay tir 0 téi il thi ta da ching minh duge mot ding thite khé. that vi boa LAC 0 Cp. Bai toan 5 Ching minh ring udin € N thi Oh, = (C8)*+(CR)? +--+ (C8)?. Léi gidi. Ta dém s6 céch chon n phan tit tt mot tap gdm 2n phan tit theo hai céch. Céch thit nhét: méi lan chon ran phan tit, khi d6 86 oéch hién nhién 1A C,. Céch thit hai: truéc hét ta chia tap 2n phan tit thinh hai tap con, m6i tap gdm n phan ti; sau d6 chon tir tap con thi nhat & phan ti (c6 C* céch chon) vi chon tit tap con thit hain—k phan tit (c6 Op-* = Cx cach chon), ta s8 c6 (C#)? cach chon; cuéi cing cho & chay tit 0 tdi n ta duge téng s6 céch chon can tim 1 (C2)?+(Cl)?+----+ (C8)? Tir d6 ta 06 dpem. Bai ton 6 Chting minh ring vdim,n € Z+, m> k thi Cran = Cn On + Or Ons 2 + C8, Chay 59 Lai gidi. Ta dém 36 céc bo s6 nguyen T = (a1,02,...,amsngi-t) VOi1 Crane is is Cée lug thita cia 2 goi ta lien tudng dén s6 dén sb tap con mét tap hgp. ‘Trong céc tap con cia tap $ = {1,2,...,.m+n+1}, théy 06 C#,,2"-* tap dang {01,02,.-. anti}, (1 <é< m4 1) trong d6 a < az < ++ < any Worn =n tha d V6i 0 < k 1) chit s6 théa man céc diéu ign: i) N gm ode chit 86 {1,2,4,5} va hiéu 2 chit 36 lién tiép luon lon hon 1 ii) N chia hét cho 11. Bai tap 6 Dat Tinh 62 my ' i prasanioeriinitonerns ites rw wis Sith Phuong phap thiét lap hé thitc truy héi trong té hop Huynh Tén Chau Mot trong nhtng phuong phap o6 higu qua dé gidi bai todn td hop la thiét lap he thie tray héi. Noi dung co bin cia phuong phép nay nhu sau: Thay vi ta dém trye tiép J(n) theo yeu cu bai toén, ta sé thiét lap he thie quan he gita f(n), f(n —1),... d tir dé tinh duge f(r). Bai todn 1 (Bungari - 1995) Cho sé nguyénn > 2. Hay tim s6 cdc hodn vj (a1, 02... ,an) ctia 1,2,...,m sao cho tdn tai duy nhét mot chi 56 i € {1,2,...,.n— 1} théa a; > ais. Bai gidi. Goi S,, 1A s6 céc hodn vj théa diéu kien bai todn. Dé ¥ ring s6 céc hoén vj mA nla Sq1. Con 86 céc hodn vj (a1,a2,...,dq) Vi aj =n (1 1). Ré rang a; An—1 (i= 1,2,...,h) nén s6 céc tap con nhu vay la: |Sy_o| + 1. Do vay: [Sa] = [Smal + |Sn-2| +1 Voi chit ¥ |Sp| = 2, [53] = 4 ta cé: tS Mat khée 86 tap con khong réng ofa tap {1,2,...,n} 18 2" — 1. Vay 86 tap con ma trong méi t@p con khong ¢6 2 phan ti nio la hai s6 nguyen lign tiép 1a: PA 4) 2 Bai ton 4 06 n qué bong biyba,...,by vd In hép hy,ha,...hon. Biét ring qué bing by (i = 1,2,...,n) chi bd vdo duge ofc hop hi,ha,...,ha:. Héi cb bao nhieu cach bd K(1 1 thi sinh ngdi trén mot bai trom, Héi cd bao nhiéu ctich phat dé sao cho hai thé sinh ngéi canh nhau luén cé dé khac nhau, biét ring trong ngan hang dé 6 ding m (m > 1) dé va hién nhién méi dé cé nhiéu ban. Bai giai. Nhan zét: Do thi sinh ng@i theo vong tron, nén mot cich tu nhién ching ta nghi t6i vige tim cdch "cit" vong tron thinh hang thing. Céch 1. Ky higu Py 18 s6 céch phat dé hgp Ig cho n hoc sinh a1, a2, ...,aq ngdi theo vong tron (mot céich phat dé duge coi 1a hgp 1é néu méi thf sinh duge nhén chi mot dé va hai thf sinh bat ky ngdi gin nhau thi nhan duge hai loai 48 khéc nhau) Ta viét a; = a; (i j) néu a; va ay cing logi dé va a; # a; trong trutng hop ngudc lai ta ching mink: Pasi = (m~ 2)Py + (m~1)Poa @ Xét mét céch phét dé hop le cho n + 1 thf sinh ay,a9,..., 41. Néu ay # ap thi bd an41 di ta c6 mot cach phat dé hop lé cho n thi sinh (a1, @2,..., a), va 06 (m — 2) céch phét d8 cho an41 Néu a1 = ay thi ta b6 Gn4s Va dq di ta c6 mét céch ph&t dé hgp 1é cho (n — 1) thi sinh (41, a2,...,@n-1). Ta ¢6 (m— 1) céch phét d8 cho (aq, @n41) dé hop 18 voi an = a). Vay ta 06 duge (1). Mat Kkhée dé thay: Pp = m(m~ 1); Py = m(m—1)(m 2). B&ng quy nap ta ching minh duge: P, = (m — 1)" + (m—1)(-1)", C&ch 2. Bai toan tuong duong véi viée dém s6 cée day (a) (i = 1,2,...,n) thda: a; € M = {1,2,...,m}Wi=1,2,....n va ay f 02,02 £ G3,--. 50a #2 ‘Trong tap hgp céc day (a;) théaa; € M, Vi =1,2,...,nva.a, # a2, a2 ¢ a3,...50, # a1, B01 An, By lin lugt la tap hop céc day (a;) mA a, # Gn VA a; = an, Do v6i méi day thude By, néu bé di s6 a, thi ta duge mot day thuge A, nén [Bal = [An]. Mat khée, dé thay |An| + [Bal = _m(m —1)"-! (do a, ¢6 m céch chon, aj41 ¢6 (m ~ 1) céch chon khée a; v6i moi i= 1,2,...,n— 1) Vay |An| = m(m — 1)" ~ [Anal ‘V6i chit ¥ |Ay = m(m — 1) ta duge: |An| = (m— 1)" + (m—1)(-1)". D6 chinh Ia dép s6 can tim, han zét: Thue chAt c& hai loi gidi JA nhu nhau. Nhung voi 18i gidi 2, "dudng nhu" te di hinh thanh nén mot huéng giai méi. Dé th4y rd higu luc cia hudng méi ndy, ta dén v6i bai tofin sau: Bai todn 6 (IMO - 1979) Cho A vd E la hai dinh déi tam ctia mot hinh tam canh déu. C6 mét con éch bét déu nhdy tit A. Tai bat cit dink nao trit E, éch c6 thé tdi mt trong hai dinh ké. Néu éch nhdy tdi E thi né ditng lai 6 46. Goi ay la sd dudng di phan 65 biét ciia ding n bude nhdy dé éch nhdy tx A dén B. Ching minh ring: Bart =0, Oma al(24+ v9)" -(2- vA] Bai giai. Ki hiéu céc dinh nhu hinh ve. ans = 0 1A hién nhién. Goi b, 1a s6 dung di tit C t6i E qua n buéc nhay. Qua 2 bude diu tién, éch c6 thé v8 A ho&e dén C hoke G. Do do: Gan = 20-2 + bana Vn > 0 (1) Tit C (hode G), sau hai buée nhay éch c6 thé vé lai C hose téi A, néu n > 2, do dé: an = Qan2 + Om-2 Yn >1 (2) Tit (1) va (2) suy ra: Gan = 402n—2 — 2dan—4 Cang vi a2 = 0, a4 = 2 ta dé dang c6 diéu phai chttng minh. Bai toan 7 (Dy tuyén IMO - 1996) Cho bing 6 vudng n x n (n > 1). Hoi 0b bao nhiéu céich dénh ddu cdc 6 vudng trong bing sao cho trong méi hinh wudng 2x 2 cb ding 26 vudng duoc dank déu. (Hai cdch dank déu duge coi la hdc nhau néu c6 mét 6 vuéng no dé ma trong céich nay thi duge dénk déu cén trong céch kia thi khong) Bai gidi. Goi S,, 1a s6 céch dénh du trong bing n xn. Xét tap T gdm che 6 vudng nim trong ct cuéi cing (tinh tir phai sang) va hing cuéi cing (tinh tit trén xudng), ta Boi A, 1A céc céch dénh déu ma o6 hai 6 vudng ké nhau trong T cing dutge dénh déu hole ching khéng duge dénh dau va B, IA cée céch dénh déu ma céc 6 vudng trong T duge dénh diu xen ké. Dé théy mai céich dénh déu thude By sé img véi mot céch dénh du thude By, con méi céch dénh déu thuge A, sé tmg véi mot céch dénh dau thudc Ani vA mot céch dénh dau thudc By. (Digu nay suy ra khi xét bang 6 vudng (n — 1) x (n— 1) 06 duge tix bing n x n sau khi bé 7) es f ‘ f Tir dé ta c6: [Bul =|Bn-al, [Ant = [nal +1Ba-il. (n> 2) Ma Sp =|An| +|Bql Vn > 1, nén S, = 254-1 — Spa Wn > 3. Dé thay ring: Sp = 6, Sj = 14. Tw d6 bang quy nap ta cé: S, = 8n —10, Vn > 2. Bai toan 8 Tim cic sé nguyén duong n théa man: i) n 6 1000 chit sé. ti) Tat cé cde chit s6 cian la ie, tii) Hiéu ctia 2 36 lién tiép bat ky cia n ludn bang 2. Bai gidi. Trong tap hop Si céc s6 nguyén duong n c6 k chit s6 théa (ii) va (ii), goi At, Br, Ck, Di, Ex lan hot 1a tap hop cée s6 tan cing bai 1,3,5,7,9. Tix méi s6 thude Ay néu ta bd di chit s6 tan cling thi nhan duge mot s6 thude Bea, mat khdc tit méi s6 thudc By-1 néu ta bé sung thém s6 1 lam chit s6 tan cing thi nhan duge mot s6 thuge Ag, do dé |Ae| = |By-r|. TH mbi s6 thuge By néu ta bé di chit s6 tan cing thi nhan duge mét s6 thude Ay; hoc C1, néu ta bé sung them 36 3 lam chit s6 tan cing thi nhan duge mot s6 thude By, do a6: |Be| = |Ax—al + |Ck2| Twong tut |Cx = |Be-a| + |De-al, [Dil = Ceal + [Baal va [Bx Sit dung 5 ding thie trén, bing céch thé lien tuc, ta 6: [Daal (Vk > 1) [Si] = |e] + [Bel + [Cel + [Dal + [Zed = |Aeal + 2]Be1] + 21Ceal + 2)Dy-al + [Bea] = Aga] + 3|Bx-2| + 4|Cx-2| + 3[Di-2] + 2|Ex-2| = 3]Aj-a| + 6|Ba-s| + 6|Ci—s| + 6|De-s| + 3121-3] Suy ra: [Se] = 3]S,-2 (Wk > 3). Do [52] = 8 nén |Siooo] = 8.3! ‘Tuong ty nh trén ta c6: Bai toan 9 Tim sé cic bd sé nguyén (a1, a2,...,dn) (n> 1) théa man: ail <1, Vi=1,2,...,0 lax—aiyal <1, Wi=1,2,....2-1 Bai gidi. Trong tap S, gdm céc bn s6 nguyén (a), a2,...., an) tha d& bai, goi An, Bas Cn 1a tap hop ee bd ¢6 a, bing —1,0,1 tuong ting, Ta c6 ngay: |S,] = |An| + |Bn| + [Cal M&t khéc, dé thay tix mdi bé thudc A, hoe B, ta c6 thé bé sung Qny1 = —1 dé duuge mot b6 thude Ans nén |Ansil = [Anl + [Bal Titong tu ta c6: |Ca4i| = |Cn| + [Bal va [Basal [nl + [Bal + 1Cn| Spl. 67 Tir d6 ta c6: [Snot] = [Antal + [Basa] + Crs] = (|Anl +1Bal +[Cal) +1 Bust + | Bal = 2|Sn] + [Spal . Két hgp v6i |S2| = 3, |So| = 7 ta tinh duge: joy) =< GEVO + = vIH uo Re Tacé thé phét biéu bai todn dudi dang phite tap hon, ching han: Bai toan 10 Tim sé cde da thife P,(x) bac n chén théa man céc diéu kién sau: i) Cée he 86 cia Pa(x) thudc tap {0, +1}. ii) Ton tai da thite Q(z) ciing cb céc hé 36 thude tp {0,41} sao cho P,(z) = (#7 -1)Q(z) Cu6i cing xin néu ra mot s6 bai tap dé ban doc rén luyén. Bai tap 1 (VMO - 1997) N dudng tron chia mat phéng ra lam bao nhiéu phan néu b&t cit cap hai dudng tron nao ciing cit nhau tai hai diém phan biet va khong c6 ba dudng trén nao 6 giao diém chung. Bai tap 2 Cho n la sé nguyen dung. Dat $ = my + cay2 +--+ + tat trong dé iy: (@ = 1,2,....n) nhdn gid tri 0 hode 1. Goi A(n),B(n) lan lugt la sé 2n-b6 (2iy-+ stn thy --1Yn) €6 Sle v8 chin, Ching minh: A(n)_ (2"~1) (—) ay Bai tap 3 Ky higu M = {—1,0,1}. Tum 36 cdc b6 (a1,...,an) théa: 1. a; thude M v6i moi i= 1,2, 22a; —ai41 thude M véi moi n. s2ye.yn 1 Bai tap 4 Cho trudc 36 nguyén duong n. Xét tat cd cdc téng S=ay ttayat---+2nYn vdi x,y € {0,1}, i= Tn Goi Sp la s6 téng lé, Py 1a 86 tong chin. Ching minh: Sa 1 PPS =m Y tung giai va su tudng minh Idi gidi qua mét sé bai todn té hop Lé Van Quang ‘Trong céc ky thi IMO, céc bai toan té hgp (Combinatorics) duge dt trutde dau bai bing chit "C". Vi du: Bai 6 IMO 2005 do Rumani dé nghi duge ghi: C6(ROM). Khi doc dau bai ctia cée BTTH (Bai tosn té hp) thi hoc sinh déu cé thé hiéu céc gid thiét va két uan kha dé dang, nhung gidi duge chting IA diéu kh6 khan. Tix bang két qué diém cho thay s6 hoc sinh gii duge diém t6i da rét it, didu dé chiing t6 day 1A loai bai todn kh6, tham chi c6 trong tay 1d gidi clia téc gid ra bai ton dé thi khong phai hoc sinh nao cing hiéu day dii va cin ké ldi gidi. Ning ngudi ty gidi bai toan dé bing mot céch khée thutng hiéu dugc Idi gidi cia téc gid mot céch khé dé dang. Tai sao lai nhut vay? Mot sé BTTH thutng dé cAp mot s6 yéu t6 rang budc theo nhing quy tée nto dé. ‘Yeu cdu cita bai todn 1 dénh gié mot dai lugng nado dé lién quan dén céc yéu t6 di dé cap, hoke ching minh m9t quy t&e n&o dé ludn thye hién duge, hotic ching minh mot quy lugt mo d6 nghiem ding. Latoc dé tyr nhién dé tiép cén vige gidi loai bai tofn nay da duge hinh thanh cho hoc sinh tit cée lop dui gdm cée bude: 1. Chon an dé mo t& céc yéu t6 trong dau bai think mot phuong trinh, mot bét phuong trinh ho&%c mét hé hén hgp chita an da chon. 2. Xitly céec diéu vita mo ta theo yéu cu ciia bai todn bing céch giai ra nghiém hote bién déi thinh nhimg két qua gitip cho vige hinh thanh quy téc hay quy luat théa yeu cdu bai todn. ‘Tir ¥ tudng gidi nhu thé thi khau then chét nb&t 1a thé hien tuéng minh ra mot loi gidi cu thé. Do bai todn khé, ngudi gidi duoc bai toan chic chin phai chi ra duge méi quan hé ndi tai cla céc yéu t6 trong bai toén thong qua cdc ky nang bién déi tinh x40 hoke nhiing nhan xét tinh té, bin chét nhét tix he da mo ta dugc. Ngay 6 buéc 1, vige khéo chon dn, ho&c dit thém dn phy hoe tich hgp cde yéu t trong dau bai... IA sy séng tao rét c& biét riéng cia ngudi gidi. Hoan thanh buéc 1 da 18 mét thanh cong ma khong phai hoc sinh ndo cing lam tét, nhung diéu cét yéu la xit 1y thanh cong d bude 2. Trong bude nay thing nay sinh mét 86 vin d8 1a céc két qué thu duge thug 1A do cée phép bién déi hé qua. Vige khdo sét nguge lai 1A can thiét, hokc it ra gidi quyét duge vin dé tdn tai tinh huéng ma da chi ra, Dua ra mot vi du cu thé dé ching td tdn tai tinh huéng cing khong phai d@ dang, 69 dn tao duce mot quy trinh hgp ly, chit ché, o6 he théng dé xay dung duge tinh huéng 6i lic lai khé hon yéu cu cia diu bai. Céc BTTH nay déu do céc nha toén hoc ling danh trén thé gidi séng téc nn trong Tai gidi cia ho thudng thong béo mot khém phé méi vé tri thttc todn, mot "bit bién" no dé, hoge kién thiét mot thuat tosn nao dé... Doc cée Idi gidi cia ho, hoc sinh hoc tap duge nhiing céch dat vin dé mot céch sdng tao, nhitng ky ning bién déi dieu Tuyén béc thay, nhiing hogt dong vé tich hgp céc dit kign rigng 1é thanh nhitng két qua su sé ma tir dé 06 thé dua ra nhing két luan x4c dng, céc khéing dinh ma ho thiting dat ten 1a cde "bé da". Cing ¥ tung gidi nhung c6 thé c6 nhiéu céch dé thuc hign su tudng minh Idi gidi, Trong 6 c6 Idi gigi mA hoe sinh cho 1a kh6 hiéu. Viee goi ¥ cho hoc sinh mét tung gidi VA dong vién hoc sinh né Iyc thyc hign theo eéch cia minh, dé tuéng minh ra mot Ji gidi cu thé cho BTTH, phai chang la.céch hop ly dé gidp cho céc em hoc sinh mdi bat du lam quen véi céc BTTH héc bite nay. Sau day 1A céc bai todn minh hoa cho eée diéu vita dé cap. Ba BITH duge chon minh hoa nim trong céc ky thi: Chon hoc sinh gidi Quéc gia 2005 cita Viet Nam, USAMO lan thtt 30 va, IMO 2005. Chiing ta s8 xem xét ching & hai Khia canh: ¥ tung gidi va su tudng minh Idi gidi. Bai toan 1 (HSGQG 2005, Bai 3) Trong mat phiing, cho bat gide Idi AiArAaAAsAgArAg ma khong 06 ba dudng chéo nao cia né cét nhau tai mét diém. Ta 901 mbi giao diém ciia hai dudng chéo ciia bét gide la mot mit. Xét cde tit gidc Idi ma méi tit gide déu c6 ci bén dink la dinh cia bat giée da cho. Ta goi méi tit gic nhu vay la ttt giée con. Hay tim s6 nguyen n nhé nhét c6 tinh chat: cb thé 16 mau nit sao cho vdi moi i,k € {1,2,3,4,5,6,7,8} vai A k, néu hy higu s(i,k) la 86 tt gide con mhan Ay Ay lam dinh va déng thoi cé giao diém hai dudng chéo la mét nit da duge +6 mau thi tht cd cho gid tri (i,k) déu bing nhau. Giai. Goi n 1a s6 nguyén nhé nh&t thod bai todn. Ta cé 3(i,k) = 9(1,2) voi moi i,k € {1,2,3,4,5,6,7,8} vai Zk. Do mét mit tuong ting v6i C? c&p dinh nén: nC} = > s(i,3) = O3.s(1,2) 4 3n = 14s(1,2) 1S Suy ra n chia hét cho 14. Ti dé: n > 14. C&ch t6 mau 14 niit thod man bai todn sau: {1,2,3,4}{1,2,5, 6}{1,2,7, 8}{2,3,5,8}{2,3,7,6}{3,4,7, 8}{3, 4,5, 6}{1, 4,8, 5}{1, 4, 7,6}5,6 Nhfn xét. 70 i | } i { | | | 6 i f ~ Voi mot hinh lép phuong c6 thé ghi lai m@i dinh mot s6 chon trong tap {1, 2, 3, 4,5, 6, 7, 8}, hai dinh khée nhau ghi hai s6 khéc nhau. ~ MBi canh hinh lép phucng c6 thé tuong ‘ing v6i ding 3 canh song song véi n6. ~ Méi dutng chéo cita mat hinh lap phuong c6 thé tuong ting voi ding 3 dutng chéo cia mat cing nim trong mat chifa n6 hoc trong mat déi dién véi nd. ~ M@i dudng chéo (chinh) cia hinh lp phuong 6 thé tuong ting voi ding 3 duing chéo (chinh) cdn lai. V6i ¥ tudng trén c6 thé hiéu ly do tai sao lai chi ra ditge céch té mau nhu trén, Bai toan 2 (USAMO 2001, Bai 1) Cé 8 cdi hop, méi hop chita 6 tréi bank. Tum 36 mhé nhét sao cho mdi bank tu) y déu duge to mot trong n mau thod man déng thoi hai diéu kign sau: 1. Trong méi hép, khong 6 hai bank nao duge t6 cing mot mau. 2, Hai hop bét ky) cb chung khdg qué mot mau. Giai. +) Goi a; 1a sé mau xuft hign i lin. i= 1,2,...,4, (k 1 ta 06: ae 1a it GOP = G ape 1 L&y (1) tri z (2) réi cong véi 30) ta duge: po da + Eo aR ee Tir do: 7 +) Céich 16 sau cia 23 mu théa bai toén (goi tén mau la: 1,2,...,23) HopI 13 4 5 6 7 HopIl 1 8 9 10 11 12 Hoplll 1 13 14 15 16 17 HopIV 2 3 8 13 18 19 HopV 2 4 9 14 20 21 Hop VI 2 5 10 15 22 23 Hop VII 6 11 16 18 20 22 Hop VII 7 12 17 19 21 23 Nhan xét. +) O hinh duéi, méi duémg tuong trung cho méi hop, cée giao diém 6 trén dudng tugng trung cho céc banh. +) C6 dang 8 dudng, méi dutng chita ding 6 giao diém va cé tat cd 23 giao diém. Hai dudng bat ky c6 téi da mot diém chung, +) MBi céch dénh s6 23 giao diém, ti 1 dén 23, cho ta mot céch t6 mau trén cdc banh 6 8 hop théa céc digu kign bai toa, Bai toén 3 (IMO 2005, Bai 6) Trong mot hy thi hoc sinh gidi, cic tht sinh phai gidi » - a 2. A 6 bai tovn. Biét ring wii hai bat todn bt hj luon od niu hon = 6 tht sinh dy thi, gid duge od hai bai todn nay. Ngodi ra khong oé tht sinh ndo gidi duge od 6 bai todn. Ching minh rang od it nhét 2 thé sinh sao cho méi ngudi trong ho gidi duge diing 5 bai todn. Giai. +) Goi n la s6 thi sinh tham gia ky thi vA ts IA 86 thf sinh gidi dugo dling k bai toam (k= 0,1,2,3,4,5,6). Ta o6: t= 0; N= ao +4 +22 +49 +44 + os. (4) Ta cd chting minh: 25 > 2 72 Speech ne rr pe vai aataalcH i; +) Voi i,j vai Aj, goi 9(i,j) = s(7,2) 1a 96 thf sinh gidi duge c& bai i va bai 5,(i,5 = 1,2,3,4,5, 6) ‘Theo gia thiét lun c6: 5s(i,j) > 2n. Do d6: 5s(i,j) > 2n +1. Co tht c& CF = 15 cap (i,j) mA i < j nén: 5s, 5) > 15(2n + 1). roa Do d6: 5S => s(i,7) > 32n +1) (5) tg +) Ta cing 6: S = Ct, + Chas + Chg + Obs = xp + 323 + 6x4 + 1025 (6) Te (4), (5), (6): y+ 3x3 + 64 + 10z5 > 6(Z9 + 2 +22 +03 +24 +05) +3 hay 4s > 6 + 62; + 5x2 +3z3+3>3 a) Ti do: B21 +) Ta ching minh thém zs khong thé bing 1. Gid stk 2s = 1. Lite do tit (7) cho 29 = 21 = Tit (6) cho: = =0,vaty=n-1. S=6n+4. 8) ‘Trudng hop nay c6 duy nh4t mét thi sinh A lAm duge ding 5 bai, cdn lai tt c& déu lam duge diing 4 bai. +) Goi bai duy nhét ma thf sinh A khong lam duge la bai r va & 18 86 thi sinh gidi duge bai r. M@i thi sinh trong k thf sinh nay ngoai’vige gidi dugc bai r con gidi duoc dang 3 bai nifa trong s6 cdc bai tosn cn lai nén: 6 ak= D7 str) 0 selder +) Dat 2n +1 = 5a. Ta 6: 9(%,j) > a. Néu a khéng phai 18 s6 nguyén thi: (i,j) >@ + 58(i,j) > 2n+2. Suy ra: DY 5s(i, 3) > 15(2n +2) & S>6n4+6. aaa 73. ‘Trai voi (8) +) Néu a la s6 nguyen thi higu s(¢,7) —a I s6 nguyén khong am. Ti $ = 6n-+4 viét lai Dicg(s(t, 4) — a) = 1, suy ra trong 15 96 hang 9(i, j) voi i < j, phai c6 14 s6 hang c6 cing gié tri IA a va ding mot s6 hang 06 gis tri la. a+ 1. Goi s(p,q) = a+ 1. Do d6 gis tri cia D9. jar (74) chi 06 thé 1A 5a hole 5a +1 tity theo r Khong thudc ho&e thudc {p,q}. Két hop véi (*), ta c6 hote 5a chia hét cho 3 hoke 5a +1 chia hét cho 3 (*). +) vi thi sinh A gidi duge 5 bai, nén tdn tai mot bai t khée véi cée bai p, q,r ma tht sinh A gidi duge. Goi h 18 96 thi sinh gidi duge bai t. Trong s6 h thi sinh nay, thi thf sinh A gidi duge bai ¢ va them ding 4 bai nia, vA A—1 thi sinh cdn lai ciing gidi due bai ¢ va thém ding 3 bai nia. Vi vay: ° . 44+3h—-1)= 5 s(t,3) hay D> s(¢,9) = 3h +1 sage jana Dot ¢ {p,q} nen: 6 DY s(,3) = 5a. anit Suy ra 5a = 3h +1 va Sa-+1 = 3h +2. Diéu nay mau thuiin véi (**). Qua ldi gidi bai todn trén, t6i cho ring cée bai toén sau ciing c6 cing § tuéng gidi va cc em hoc sinh c6 thé thyc tap tudng minh léi gidi. Bai todn 4 Chon la sé nguyén lon hon hodc bing 3. Trén méi canh va méi dung chéo cia n-gidc déu AyAz....Aq ngudi ta mudn ghi mot sb nguyén duong nhé hon hod bing P, sao cho cée diéu kién sau déu duge théa man: i) Moi 36 nguyén ta 1 dén p déu duge ghi. i) Voi méi tam gidc A,AjAx tiy yf déu 05 hai canh duge ghi hai s6 gidng nhau va s6 nay lén hon sé dugc ghi trén canh con lai. Hay xée dink s6 nguyen p lin nhat dé cb thé ghi céc sé théa man céc diéu kién dat ra. Voi gid tri nay cia p, hdi 06 bao nhieu céch ghi thda man diéu kién bai todn, Bai toan 5 C6 m_+2 cdi hop, méi hop chtta m trdi banh. Tim s6n nhé nhét sao cho méi bank duge t6 mét trong n mau théa man déng thoi hai diéu kién sau: 1. Trong méi hép, khong c6 hai bank nao duge 16 cing mot mau 2. Hai hép bat ki cé chung khéng qué mét mau, Bai toan 6 Trong mét hy thi hoc sink gidi, cdc thé sinh phdi gidi 6 bai todn. Biét ring 74 t 2 2 16; hai bai todn bat by) ludn 6 nhidu hon = 36 tht sinh du thi gid duc cf hat bat todn nay. Ngodi ra khong c6 thi nao gidi duge cd 6 bai todn. @. Goi k la 36 tht sink gidi duge ding 5 bai todn. Tum gid inj nhd mhét ctia k. , ly b. Chitng minh tén tai ba bai todn ma cé nhiéu hon 5% thé sinh dy thi gidi duge. . 1 © Chong mink tn ti bin bai todn ma 05 nhiéu hon = 36 tht sinh du thi gidi duge. Bai toén 7 Cho mét n-gide Idi c6 din tich S. Ching minh ring tén tai mot cank AB ctia da gidc va mot diém M thudc mién da gide nay sao cho khodng cach tix diém M 4s én dudng thing AB khong nhé hon =. Tai ligu tham khao [1] Toan hoc va Tudi tré s6 340/2005; 344/206: Bai cia Thay Vi Dinh Hoa (DHSP Ha Noi). [2] Toan hoc va Tudi tré s6 341/2005: Bai ciia Thy Hoing Ngoc Canh (THPT chuyen Ha Tinh). (3] USA and International Mathematical Olympiads 2001 Titu Andreescu and Zuming Feng. {4] http://www.mathlinks IMO Shotlist 2005. [5] IMO 2006 Solutions. | | Gidi thiéu mot sé bai toan dai sé cé xuat xt tiv hinh hoc Nguyén Dang Phat Bai viét ndy gidi thiéu voi ban doc mat s6 bai todn dai sé dac thi, cd cudt wit ti hinh hoc, ma noi dung bao gdm hai thé loai la hé phwong trinh dai 36 va cuc tri dai 36. 1 Mé6t sé hé phuong trinh dai sé dic biét co xuat xit tit hinh hoc. Trude hét, hay xuSt phat tir mot s6 hé phuong trinh dai s6 bic hai don gidn (06 xubt xif tit hink hoc). Bai toan 1. Gii va bién luan he phuong trinh sau: (a #0, b #40, c#0); Q) Lai gidi so luge 1 A Dat 7 1A gid tri chung cia ba ty s6 trong hé phuong trinh (1) réi tinh ¢ (ma ta thutng goi la dn phu) vac, y, z. Ta duge: a=t(2?—yz), b= ty? — 22), c= (2? — zy), Ti d6 suy ra (néu t < 00): abe 2(2° + y° + 29 — 3zyz); (2) va hai hé thie nita tuong tu (46i voi ¥ - ca va c? — ab), thu duge nh’ hoén vj vong quanha+b—+esavarsysz-. Cuéi cing ta di dén két qué sau day: ‘Tra 18. 1°) Néu ba s6 a,b,c khée khong di cho va khée nhau d6i mot thi 2,y,z cing vay 16 (c #0, y #0, 2402, y, z doi mot khée nhau). Khi 6, ¢ cing khée khong va hé phuang trink (1) la v6 dink), 06 biéu thite nghiém nhw sau: z y Feet Fog HA 6) 2) Néu a = b =c thi he phuong trinh (1) cing 18 vd dinh, vi khi d6 x = y = z va ly gid tr] thy J, ké cA 0. Nhan xét: 1°) Biéu thitc nghiém ciia hé (1) 6 tink ch&t d6i xing, dutge suy ra tit (1) bing céch thay cdc hing s6 a,b,c lin lugt béi cée dn x,y, 2, va nguoc lai, thay x,y,z bai a, b,c tuong tmg. Ta néi ring he (1) c6 tinh chit déi hop. 2°) Tit 16i gidi ciia he (1) ta dé dng suy ra hai he phuong trinh sau (hé ba phuong trinh ba dn) c6 nghiém duy nhét: a oy ec! qa’) ntytz=atb+o (abc#0) e p—2x @ 6 (”) xyz = abe; (abe ¢ 0) Bai todn 2. Gidi he phuong trinh (An 18 z,y, 2): z-a_y-b_z-c¢ @ —bo d=e = (t-a) + (y—B)? + (2-0)? @) =a Déng thdi, ching minh ring: Néua =b = =0 va gidta a’, U’, ¢ 06 he thiée A(a? +8? + 6?) + (pa! + qb'+ rc) +B =0, (i) (trong 46 A 40, BAO; a fa, VAL, ¢ Ac; PAO, g £0, r #0) thi git x,y,z cing c6 hé thifc sau day: B(x? +4? + 2”) + Ape + qy+rz)+A=0, (ii) Huéng dan. Dat ¢ 1a gid trj chung cia ba ti s6 4 v6 tréi cia (*), cing c6 nghia dat t= (e— a) + (y—6)? + (z—c)? la dn phy. Latu 9 ring dat dn phy nhu thé ndy cing 1a phuong phép chung dé gidi he phuong trinh dai s6 c6 dang (*) nhu he phwong trinh trén day. Sau khi thuc hién mét s6 phép bién déi dai s6 tuong duong ta thu duge dép 86 cia bai ton, Phan nay danh cho ban doc tu kiém nghiém. ‘Tra 18%. Bidu thitc nghigm cia he c6 thé duge viét (sip dat) dudi dang sau day: ad-a_Y-b c¢-c ta y-b z-e = (a =a $Y bP + (6-0; c+) 7 i i han xét. Biéu thite (**) cia hé (*) c6 tinh chat d6i xing cing giéng nhu biéu thite nghiém (3) cia he (1) duge chi ra trong bai toan 1 é trén. Cu thé la (**) duge suy ra tit (*) bling céch gid nguyén cfc hing s6 dit kién (44 cho) a,b,c nhung lai thay céc hing s6 a’,¥,¢ lan Iugt béi céc An x,y, 2 vA ngude lai, thay +, y, z lan lugt béi a’, Wc. Xuét xf cia bai ton 2. Ti bai toan dai s6 1 trén day, nhan nhan xét vé tinh chét, "déi lap" cha biéu thitc gitta nghiém ("An") va hing s6 dit kién, tac gid bai viét nay lién tung dén tinh chét "46i hop" cia phép bién hink nghich dao trong mat phiing cing nhu trong khong gian. Trude hét, viét phyong trinh vécto ciia phép nghich déo trong khOng gian cyte O va phutdng tfch p =r? = 1) rdi da xudt thanh bai todn 2 trén day. OM.OM' =1 (OM 770M) & OM = 2 hay IA: Sau 6, viét phuong trinh vécto cia mat ciu (hay duéng tron trong m&t phing) AZ? 420.7 +B =0ri xuat bai todn chtmg minh tiép theo (Phép nghich dio bién mat cu thanh mat cdu hay mat ph&ng). Bai ton 3. Giai va bign luan hé phuong trinh (An z, y, z): (a — x) cosatan? a + (y — b) cos 8 + (z —c) cosy (x — a) cosa + (b— y) cos f tan? B + (z —c) cosy (x — a) cosa + (y — 8) cos 8 + (c — z) cosy tan?y = 0. cn) (cosa #0, cos £0, cosy £0; ee) . 1 : 2 =, tan?g= 1, ton? a Hung din. Thay tanta = > ~1, tan? = a5, tanty = a 1 vio he phuong trinh (***) thi dua duge he nay v8 dang sau day: an fe oo (x a) cosa + (y ~ 6) cos + (2 - ¢) cosy = as = cr) ‘Tra lWi. 1°) Néu o,f vay khong c6 méi lién hé gi véi nhau, cy thé Ia: cos? «+ cos? f+ cos? # 1 thi hé phuong trinh 6 nghigm duy nhét z= a, y=b, z=c. 2°) Néu cos® a + cos” 9 + cos? y = 1, hé phuong trinh c6 v6 s6 nghiém, dang: r=a+ Acoso, y=b+AcosB, (trong d6 1A mot tham sé tiy ¥, 4 € R) z=ct eos, Xuét of ctia bai todn 3. Bai toén trén day duge phét hign dong thdi véi bai toén 4 (duge trinh bay ngay sau day), ndy sinh tis vige dat bai todn 4. Bai toan nay cé nhién 8 dé han nhiéu so véi Bai todn 4. Nhan xét: Ching ta c6 thé dé dang mé rong bai todn cho trudng hgp nhiéu dn: n an 21, 2,-.+4q, dOng thoi cho n gOc a1, 02)... an. Bai todn 4 cing vay. Bai toan 4. Gidi va bien luan he phuong trink (dn 1a «, y, 2) ane _b-y_c-2__ ple? ty? +22) + g(zcosa +ycosf + 20084)? cosa cos cosy (p+ g)(acosa+ y cos B + 20087) Q (a) trong d6 p #0, ¢ #0, P+a40, 00); (i) M Yetta aye Pare (ii) théa man he thite (iii) sau day: Ely? + 2? — 23)2? = OP + 2 — a?)a?; (iii) Chi thich: Khong cn phai gidi he phuong trinh 2) Dio lei, néu 2, y, 2 1A ede nghiém duong cita he phutong trinh (2) (gdm 2 phuong trinh (ii) va mot phuong trinh (iii): Ha = {(it), (#i)} thi cing théa man he thie (i) ‘Tom Iai Fa{(@), (ii) > Ha{(éi), (ei)} 3°) Ching minh ring eéc bé 1; (j = 1,2) ¢6 nghigm (duong) kkhi va chi khi céc s6 duong a,b,c da cho théa man cée bat ding thiic sau day PKAPre);, P 0): om P+y+ 24 Pad Pe2¢e (a>0,b>0,c>0,d>0); (*) Pega ye R= 2+ PoP 4a; (+#) Xuét xf cia bai ton 8. Bai toén 8 trén day c6 xudt xt tir bai toan hinh hoc khéng gian sau day: Bai ton 8’. Cho tit dign gin déu ABCD (ma ti dign d&u Ia trutmg hop d&e biet), nghia I8 cée canh d6i dign bing nhau: BC = DA,CA = DB, AB = DC. Hay xéc dinh céc khoang céch tit mot diém P da cho dén céc dinh cia ttt dien biét ring diém Q, déi ximg véi P qua tam O cia mat cdu C(ABCD) ngoai tiép tt dign di cho c6 khoang céch dén céc dinh A,B,C,D cia ttt dign theo tht ty bang QA =4,QB =b,QC =¢,QD =d. Huéng dn. Ap dung dinh ly vé dutng trung tuyén trong tam gid vA tinh ch&t cia trong tam ctia mot hé diém. Tuy nhién cting cé thé sit dung phuong phép vecta, Dap 86. Gidi he H{(*), (*)}, ta duoc két qua: 1 1 HOFAtP a), Poet e+a—W), 2 1 FOrPP-A, Pater e yea) (***) Bai todn luén c6 nghiém (duong) duy nhét, xéc dink bdi céc hé thite (***). Muén vay, hay ching minh tinh chat sau day, d&c-trung cho mét tit dién gan déu (bao gm c& ti dién déu), tuong ty véi dinh ly Pompiu trong hinh hoc phéng, déc trumg cho mot tam giée déu. Dinh ly Pompiu (trong hinh hoc khong gian) c6 noi dung nhv sau: Binh phutong céc khoang céch tit mot diém P bat ky trong khéng gian dén cdc dinh cia mot tt dién gin déu ABCD da cho luén biéu thi dign tich céc mat cia mot tit dién (7) nao d6. (Digu 6 c6 nghia cy thé nhw sau: Binh phutong khoang céch tit P dén mot dinh ndo d6 cia tit dign gan déu ABCD di cho khong 1én hon tng binh phuong cée Khodng céch tix P dén ba dinh can Iai). 82 Tit dién (7) nay suy bién thanh mot tt diém phAng (tite 14 c6 thé tich v(T) = 0) khi va chi khi diém P tring véi diém xuyén tam déi cia mét dinh nado dé cia ttt dién gan déu da cho ABCD trén mat cu C(ABCD) ngoai tiép tut dien 6. Hu6ng dan chiing minh dinh lf Pompiu trong hinh hoc khong gian. Sit dung vecto va tinh chét cite tam ti cy cia mot hé diém, chimg minh cée bat ding thie hinh hoe sau: PA? < PB? + PC? + PD?, va ba BD.T tuong tt Q) PB? < PC? + PD? + PA?, 2) PC? < PD? + PA? + PB, (3) PD? < PA? + PB? + PC?. (4) Dfu ding thite xdy ra khi va. chi khi: P € {4’, B,C’, D’}, trong d6 A’ = (E9(A), B’ = €o(B),C’ = E(C) va D’ = o(D), O A tam ciu C(ABCD), tiy theo P tuong tng thudc vao bit dang thitc (1), (2), (3) hay (4) Cha thich bé sung. Trong muc 1.2 trén day, tac gid bai viét nay dA gidi thigu them 4 bai todn méi cing vé gidi nhing hé phuong trinh bac hai c6 3 hode 4 an, trong dé ¢6 chi 16 xuit xit hinh hoc d6i véi ba bai todn 5, 7, va 8. Chinh viée giai céc bai todn nay, céc ban da cho 101 gidi dai 36 cita céc bai tofn hinh hoc 6’, 7’ va 8’, 1a xuét xit hinh hoc clia céc bai todn dai s6 5, 7 va 8. Ngoai ra, téc gid cée dé ton nay ciing dé nghi ban doc tim tdi them 1di gidi thudn tity hinh hoc ctia céc bai todn hinh hoc xuAt xtt 5', 7” va 8, g6p phn lam phong phé va da dang cho Idi gidi céc bai ton 46. D6i véi bai toén 6, tée gid cia né cing xudt phét tir mot bai toan hinh hoc ma dé xudt, nhung lai khong gidi thieu trong bai viét. Tée gid c6 dung ¥ dé lai, danh cho ban doc phan dodn, suy xét vé ngudn g6c rudt aif n&o tit hinh hoc cia bai todn dai s6 (bai 6) may. Tuy nbién, xin luu ¥ ban doc 1& vige phén dodn vé zudt att hinh hoc cia bai toén 6 thyc ra khéng cé gi Ad Khan lm. Cc ban chi cin quan tam dén ¥ nghia hinh hoc cita céc dang thie (ii) va (ii) cing nhu ¥ nghia hinh hoc cia hai bd s6 dutong (a, 6, c) va (a,y,2). Tu 46 ban dé dang chi ra déi tugng hinh hoc khéng gian nao dn 6 ding sau 6 con sé, trong d6 a,b,c la da cho con 2, y, z 18 cdc dn s6 duong cén tim. Sau cing, xin gidi thigu hai bai todn vé hé phwong trink dai sé bac hai phitc tap han va chtta nhiéu an (5 hoc 6 an). Bai todn 9*. Gidi vd bign luan he phuong trink bac hai ( séu dn x:,ye,i = 1,2,3 a nhitng s6 duong): wy [2tusmtwantn=d (*) apt yt + kaa = 23+ yf + koays = 23 +98 + hesys = C7; (#4) trong 6 c va dla nhiing s6 duong va k € R; c,d, k cho trudc, a) Lai giai thw nt: Lai gidi gém hai bude sau day: 83 Buée 1: Néu hé phuong trink H{(*), (**)} 6 nghiem thi 2; = x2 = 23 (= 2) va w= t= Ys (= 9). Tit (*) thay yy = d— zs, ye =d— 2, va ys = d— 2p vio (**), sau khi thuc hign cde phép tfnh ri rat gon ta dugc hé ba phuong trinh sau d6i véi ba dn 7,22 va x3: (ker, ~ 2x9)d — (03 + kagx1) +X =0 (i) va hai phuong trinh tuong tu bing céch hodn vi vong quanh céc chi sé: 1 + 2+ 3 > 1, trong dé ta di dat X=? -2 +3 a3. luge d bing ba biéu thitc khéc nhau nhu sau day, trong dé biéu thitc sau duge suy tir biéu thite diing truéc n6 nh’ hodn vi néi tren ai tkegrs-X _ 23+kogm—X _ af-keye2—X a = Btsana# kag — 2g ‘hay — 23 kag — 2a, (ii) (v6i quy uéc néu méu s6 triét tiéu thi tir s6 cing triét tieu). Sau d6 ép dung tinh chét cita day céc ty s6 bing nhau dé khit dai luong X, ta thu duge ba ding thite sau: d=t=t=ts, trong dé = (3 = t2)(a9 + 22 ~ kar) (ii) 2(ea — 21) + k(a2 — 24) va t nhan duge tix t; cling nhw ts nh&n duge tit ty bai hodn vi vong quanh cée chi s6 1+ 2 3 ~ 1. (Lé duong nbién cing d8 nhan ra ring dén lugt ¢; thi nhén lai tit ty cing béi hodn vi vong quanh céc chi s6 nhu da néi 6 trén). Dén day tit cdc c&p ding thc t; = t; (i #7 € {1,2,3}) ta déu thu duge sy triét tiéu cia mot biéu thie c6 dang mot tam thite bac hai déi v6i k sau day: S(k) ) Ching han, tit ts = t, thi a, va 7 lin lugt déu c6 dang 18 nhiing da thiic bic 3 thudn nhét va d6i xing déi voi cde d6i s6 1, 22,7 theo nghia hodn vi vong quan cée chi s6 1, 2,3 nhu da néi 6 trén. Cu thé 1a: t ak? + Sk +7=0 a = ~ay0? ~ 230} — 2,23 + 3xy2025, ) B= 2x +a} + 2} — Bx2909) +7, (vi) = (eh + 2} +23 — eae? ~ zor} — 2529) + 209 + 2} + 2§ — 2y0? — x2} ~ 2402) (vii) Sau vai phép bién déi dun gién nifa, cfc hé s6 a, va trong phuong trinh (iv) cbn duge biéu thi duéi dang mot s6 biéu thite khdc nia, thuan tién hon cho viée xét diu cha chiing. That vay, ta 06 cdc biéu thie sau: (2 — t)(a3 — 21) 22 + (wa — ar9)?r4, (v’) 21 +92 +25) [(2~ 29)? + (xy — 23)? + (a1 —22)"] +7, 7) = (22~ 23) [208 2) + (23 —29)] + (2s ~ 22) (22 - af - 2) (vr) 84 thife ntfa cia cing nhu hai biéu thite nifa cite y nhd hoén vi vong quanh 1 > 2-3-1 nhu da n6i 6 trén. Ty dé suy ra, ngodi (vi’) thi 8 cling con cé hai biéu thite khée nita. D6i véi vige xét dau cia a, 8 va 7, khong mt tinh téng quét ching ta c6 thé gid dinh quy wdc ring: 0 < 21 < 2 < zy (hokc 0 < a < a3 < 1, howe 0 < 23 $m < 22). Ching han, v6i gid dinh 0 <2; < 22 < as ta thdy ngay ring a <0 va 72 0 cdn 6? > 0. Tit do suy ra biet s6 cia (iv) khong am: A = 6? — dary > 0. Ngoai ra, véi céc biéu thiic (v’) va (vii?) cita a vay, ta nhén ra ngay ring: a= 0 & =m = 03 Y=0 == 25, Vado dé: =0 & y= suy ra: , kéo theo @ = 0. Mat khac, dé thay: Tr A = 6? + (—4ay) = le = 0 va, 4-06 eau Béi vay, ta di dén két luan: A=0 4 2, = 22 = 23 (= 2) va do d6, kéo theo yi = yn = ys (= 9). Nhung A = 0 lai la diéu kign cin va dit dé k 1 nghiem duy nhét cia he phuong trinh HA («), (#*)} duge xét (néu hé nay c6 nghiém). ‘Tém Iai 1a; lap Iudn trén day chimg'té ring: Néw he 1 séu phuong trink {(+), (##)} 66 nghiém thi céc dn x, 22,23 bling nhau (va dat bing z), déng thai céc 4n 11, ye, va ciing bing nhau (va dat bing y). D6 la dpem. Va do dé, bai toan quy vé viée gidi mot +hé hai phuong trinh bac hai d6i v6i hai dn x va y sau day: Bude 2: Gidi va bién ludn hé hai phuong trinh (én x vay déu duong) rt+y=d, (Q) Pty? + kay = 2. @) ‘Tit (1) va (2) ta duige (2-Kay=P-2 (3) Ti d6 d8 dang suy ra: - Néu k= 2 vic # d thi hé phuong trinh {(1), (2)} v6 nghiém. - Néu k = 2 va c= thi hé phuong trinh c6 vo s6 nghiém, mién sao théa man (1). Cy thé 1a. I= d, { - (o 0va(2—k)(@ —c*) > 0. (©) ee <2, e2, ord. ® Cdn cdc nghiem duong x,y cia he {(1), (2)} 1a hai nghiem cia (5), xéc dinh béi: ford = (6%) K=F (ds (SON), Gay b) Nhan xét. Sau khi thyc hign hai bude gidi nhu da trinh bay & trén, vige gidi va bign luan hé phuong trinh bac hai véi 6 an dung 21,22, 73; y1, Ya, ys dén day da hoan t&t. Day I& mot he phuong tinh dai s6 d&c bigt, gdm ba phutong trinh bac nhat (*) va, ba phuong trink bac hai (**) ding thdi dai hdi tit c& cée An zi,y¢ (¢ = 1,2,3) déu la nhiing s6 thyc duong. Bay gid ching ta hay quan tam dén qué trinh bign ludn vé vige gidi hé phitong trinh {(1), (2)}. Trutéc hét hay dé ¥ dén gid tri dc bigt k = 2 lam cho hé phudng trinh ho&e vo nghi¢m, hogc c6 vo s6 nghi¢m. Néu k = 2 va c = d thi (2) duge ‘viét lai 1a (46i chiéu véi (1)): a +y? + zy = 2 = d? hay lac? + y? — 22ycosm = 2 (= d*) (20) Khi d6, hé thie (10) biéu thi dinh ly ham sé cosin d6i véi mot tam gide "suy bién" cb hai canh véi do dai ©,y cdn canh lén nbit c bing téng hai canh c6 do dai x va y (véi c= d thi tit (10) suy ra: s+ = c (= d). Sau nila, trong hai bit dng thiic (7) va (8) biéu thi digu kign c6 hai nghiém duong (cé nhién 1 duy ohAt) cia phwong trinh bac hai (8), ta hay dé § dae bigt dén bét ding thiic (7). Dé la bat ding thite diéu kin quy dinh méi lién he gitia hai s6 thuc duong c,d va s6 thuc k dé (5) c6 nghiém duong, cling tite Ja dé bai todn cé 1di gidi. Déi véi bat ding thtic kép (7) ta die biét chi ¥ dén bit ding thite k <2 (v6i e < d). Khi dé ta 06 thé dat k = —2cos-y (véi 0 < 7 < 1), 7 dutge hoan toan xéc dinh: +y = arccos =£ va do dé, (2) duge viét lai duéi dang: a +y? — 2zycosy=C; (c BE = CD, d.p.c.m) Bay gid ta tré lai ching minh diéu khang dinh tren day bing phuong phap phdén ching. That vay, gié sit trén cung AyB da cho ta tim duge ba diém Cy, C2, Cy doi mot phan biét théa man (*), ciing tite IA théa mén day dang thiic (1). Khong mat tinh tng quét, ta c6 thé gid dinh ring: a, < az < a3 vA do dé theo (1) thi dua dén: by < by < by; (2) Sau dé, ta dug mot g6c Cy = + ri ly cée diém Ap, As, Az, Ay tren Cz va Bo, Bi, Ba, By trén Cy sao cho CA = CBy = $; CA; = by vi. OB, = ay (i € {1,2,3}). Thé thi ta duge: AA;BiC = AABC, (c.g.0); do dé A\By = A,B) = AyBy = AB =. (3) Va tit cde bat ding thitc (2) suy ra AA;B,C nim gon trong c& hei tam giée AyB,C va AsBsC. Ta sé chitng té ring: AB, < AzBy ho&c A\B, < AsBy. ‘That vay, tit (1) ta duge bo — bs = a2 — a1 VA by — by = a5 — ap. Do d6 ta c6 AaAg = B,Bp; (Hinh 2) (4) AAs = BpBa (6) Trén hinh 2 ta duge hai tit gidc 181 ApByBy Ay va A3B3B2A, tuong img théa man (4) va (5), trong d6 hai c&p tia (A243, B,B,) va AgAi, ByB2) ddu ct nhau 6 C. Béi vay, theo bé dé 6 trén ta duge AaB, > AsBr, 6 AgBs > ArBo. @” Xét hai trutmg hop cé thé xay ra: 1°) Néu a; < by (Hinh 2) thi CA,B, < CBiA, suy ra AASB, < CAB, < 90° < AAs va do d6, trong AA; B,Ag ta. c6 AyBy > AyB, nén déi chiéu véi (6) ta. duce AiBy < AyBy (8) 88 2) Néu a; > b; thi cling lap lun tuong ty (ban doc ty vé hinh tuong ting) ta duge AVBaB, < 90° < AyByBo va do d6, trong AA;B; Be ta ob AyBp > AxBy nén d6i chiéu vi b&t ding thite (7) ta ditge: A:By < AsBs (9) Céc bat ding thie (8) hoe (9) thu duge tit gid dinh c6 cée bat ding thite nghiém ng&t (2), mau thudn véi gid thiét cia bai toén 1a c6 day ding thie (3). Mau thuan 46 ching t6 ho&c a; = a2 = as va do d6, by = b = bs, [nghia lA cdc diém C; (i = 1,2,3) tring nhau 4 mot diém Cy nao 46 trén cung tron A7B), hode it nhAt hai trong be s6 a; bing nhau (hay hai trong ba s6 b; bing nhau, i € {1,2,3}). Bay gid ta chiing minh ring: Néu hai trong ba s6 a; (i € {1,2,3}) bing nhau thi c& ba s6 a; d6 bing nhau, do dé cd ba s6 6; cing bing nhau. That vay, ching han gid sit a = ag, thé thi tir (1) suy ra by = by va bay gid thi day ding thuc (1) thu vé chi con ‘mot, dé 1a: a + by = a2 + by. (10) Bai todn quy vé chi cdn hai tam gidc, trong d6 diéu kien (*) duge thay béi (10). Sau 46 cing ching minh tuong tu nhu é phan trén, sit dung bé d8 4 néu thi chimg minh duge rang: a, = a2 = a3 va do dé b, = bz = bs, ta duge d.p.c.m. Bude 3. Rét cudc bai ton quy vé bai todn dying hinh don gidn sau day: Bai toan 9c. Tim trén cung trin AyB mot diém C sao cho: BC +CA AB =c, c vid la nhitng do dai cho trudc. Bign luan: Kéo dai tia BC vé phia C réi dung diém D sao cho CD = CA va do d6, BD = BC+CD = BC+CA=d. (Cé ding thitc nay 1a do ta gid sit ring C trén cung AyB a diém da tim duge, théa man diéu kign dat ra). Thé thi ACAD can 6 C c6 géc 4 day GDA (= BDA = 3). Te d6 suy ra D 1a mét trong hai giao diém Dy, Dz cia didng tron (B, d) tm B bén kinh d va cung tron ASB chita gée 6 = $ [c6 tam 1a trung diém Cy cia cung A7B da cho] dymg trén doan AB va nim cng hia véi AyB. 4, trong dé - Tir d6 suy ra cach dung diém C: Diém C la giao diém cia tia BD va cung A7B, trong 46 D € {D,, D2} = (B,d) 9 ABB. Dé théy ring BC+ CA = d. (Hinh 3) - Bign lun: Bai todn 6 lki gii (nghiém hinh) khi va chi khi: Fi aay 7 CoD (ua) AB=c 0) rt+yed S tt+y=d a? +y? —2nycosy ay = SF > 0(ie< d) Vay x va y 1 hai nghiém duong X;, X» clita phuong trinh bac hai: e-2 oe = aX + Tay 0 (12) Digu kign cn va di dé phuong trinh (12) c6 nghigm (nghiém duong) 1a: Biet s6 e- Psin?Z 2a cos? Z A 20 # cddsindwac e>dsind (13) DE ¥ ring A 6 thé duge viét lei nhu sau: _ 22 = (1 cosy)? 4c? fe 1+ cosy (14) ‘Ti d6 ta duge két qué (sau khi dé thay —2cos-y = k, véi |k| < 2) a 1 42 — (24+ ye {eu} = (%,%} = 5 (de (SEES (05) Két qué (15) nay tim duge phi hop véi két qua (9) tim ra 6 bude 2, tiéu muc a). e) Nhan xét va lai binh (vé hai Idi gidi ciia bai todn 9). Hai bai ton hinh hoc Ya) va 9c) chfnh 1a noi dung hinh hoc tuong ting véi hai bai ton dai s6 ma Idi gidi dug dé cp dén trong hai bude gidi (bude 1 va bude 2) lien tiép cita bai todn 9 vé gidi va bien ludn mot hé phuong trinh dai s6 bac hai. Ching Id hai phan cu thanh bai ton hinh hoc 9b — bin phién dich sang ngon ngit hinh hoc eta bai toan 9 (mét bai toén hon toan dai s6 vé hé phuong trinh) ma chiing ta cé thé cho né mot tén goi 1A "Mo hink hinh hoc" cia bai toin 9, mot bai ton vé gidi va bién luan mot he phuong trinh bac hai. No cho ta mét hinh anh true quan trong hinh hoc vat ly cia mét he phuong trinh dai s6 bac hai. Tuy nhién, méi 1di gidi cia bai tosn 9 mae dik khéc nhau vé phucng phép tiép cn nhung déu thé hién siic théi dac thi, riéng biét va. rat &n tuong cia tig Idi gidi. Néu nhu 1di gidi 1 (10% gidi dai 36) 06 doi hoi bj nang bién déi va tinh toén tinh té vi cdn phai thyc hién nhiéu phép todn nhung lai cho due chiing minh true tiép diéu khang dinh quan trong phét biéu trong bitéc 1 cia 1d gidi 90 bai todn 9 [va diéu dé cing c6 nghia I chi ra duge céch chiing minh true tiép tinh chat hinh hoc phat biéu trong ndi dung cia bai todn hinh hoc 9a)} thi loi gidi 2 (Joi gidi hinh, hoc) cita bai todn 9 lai khong doi héi tink ton nhung ddi hdi thong minh séng tao, dung thém dutng phu, hinh phy va chi cdn huy dong vén kién thtte ft di thuéc chuong trinh hinh hoc 8 cing di ding cho viée chtig minh tinh chat hinh hoc néi trén tuy 1di gidi nay lai khong thé cho duge céch chiing minh truc tiép ma sit dung phuong phép phén chiing_mot phutong phép ching minh gidn tiép_dé khng dinh tinh ch&t hinh hoc nay. ‘Trén day 1a phéc hoa déi nét vé tdng thé khi déi chiéu, so sfnh hai 1di gidi cita bai todn 9 v8 phwong phdp tiép cdn cing nh phwong phdp gidi quyét udn dé cia ting li gidi dé ma bai viét nay néu ra nhim muc dich trao déi véi ban doc nhing suy ngam, nhiing ¥ tudng dé xu&t va kinh nghiém xung quanh viéc gidi va khai théc m6t bai todn toan hoc hay (vé dai s6 cing nhu hinh hoc) nao 46. Néu xem xét ky lung, chi tiét hon thi truée hét, phai néi rang: Bai toén hinh hoc 9b) chi 1 mé hinh hinh hoc cia mét phdn bai todn dai sé 9 ma thoi. That vay, tré lai phan bién ludn cia Idi giai thit nhAt cita bai todn 9 (trinh bay 4 bude 2) ching ta nhan va ngay rng Idi gidi thit hai (Idi gidi hinh hoc) cita bai todn nay chi dé cap dén nhing gié tri cit tham s6 k théa man bat ding thite |k| < 2 sao cho ¢6 thé gan cho k yj nghia Rink hoc k = —2cos7, trong d6 0 <7 <7 18 d6 lén cia mot géc cia mot tam giée ndo d6 ma canh déi dign voi gée dé c6 do dai c va hai canh ciia géc y dé c6 do dai z vay ma tong cia hai canh dé ¢+y = d, déng thdi d > c. Nhiing gié tri cia kd day ddi héi |k| < 2 chi 1d mot tap con nhiing gié tri k théa min bat ding thitc (7) chit chuta n6i téi nhitng gié tri cia k théa man bat ding thie (8) duge chi ra day dit trong phan bien luan thudc bude 2 cita tiu muc a) trinh bay Jdi gidi tht nhit cila bai todn 9. Tée gid bai viét nay con dé trong, chua thdy duge ¥ nghia hinh hoc cia nhing gid tri k khéc, ngoai |k| < 2. f) XuAt >i cia cdc bai todn 9, 9a), 9b) va mGt hung dé xudt bai todn mdi Khac. Trong ky thi Olympic Ton quéc té lan thit 46 (46* IMO, 7/2005) té chtte 6 Mexico 6 hai bai toan hinh hoc phng, trong dé c6 bai todn (Bai 1) sau day: Trén céc canh ciia mét tam giée déu ABC sdu diém da duge chon lan lugt: Ay, A> trén BC; By, Bz trén CA; Ci,C2 trén AB. Céc diém nay la cdc dink cia mot luc gidc Wi A:AzB,B2CiCz 06 tét ed cde canh bling nhau. Chting mink ring céc dudng thing A,Ba, ByC2 va CyAz ddng quy. Chinh tix bai todn hinh hoc may cia ky thi IMO, 7/2005 vira qua ma téc gid da dé xuét bai toén hin hoc 9a), 9b) va bai todn dai s6 9 4 tren. Dé dugc thuan tien cho viée phét biéu cdc bai ton ndy duéi dang nhwr da trinh bay trong bai viét, téc gid di dat lai ‘ten nhu hinh 4 dé ghi: Tam giée déu xuét phét duige ky higu lai 18 CiC2Cs, cdn luc gide Ii duge xét ky higu I A,B, A,B, AsBs véi céc cap dinh Ba, Ag; Bs, Ar va By, A2 lin higt trén C2Cs,CaC; va CrC2. Ba duttng thing duge xét sé la: Ay Bo, AzBy va AsB1. Thé th, theo gid thiét cia bai toan va véi ky higu nhu trén hinh 4 ching ta thiét lap dugc ngay day ding thc (*) néu trong gid thiét cia céc bai ton 9, 9b) va 9a). Tuy mhién, rieng 91 v8 g6c, g6c 60° 6 cée dinh Cj (cita A déu CiC2C; va cing 1a cia ba AA:BiC)) da duge thay béi g6e 6 do lén 7 téng quét hon, mién AO <7 <7. Bai toan 10**. Gidi vd bien ludn he phuong trink bac hai 5 dn x,y, z,u,v sau day zu-yu_ aw (Ho) trong dé a,b,c,d,e, f la cdc 6 thue khéc khong da cho. A. Léi gidi "so cp" cita bai todn. 1°) Nghiém tdm thudng ciia bai todn. a) Hé phuong trinh 1, (*) 06 thé viét Iai duéi dang sau day: Néu dat \ 1a gi tri chung (ma ta thudng goi 18 dn phu cia hé) clia day 5 ti s6 bing nhau (*) thi ta c6 6 dang thite sau: ©) b) Nhan théy ring véi z, y, z,u,v tity J, bao gid ta cing c6 dong nhét thite sau day (d8 dang kiém nghiém): (az —y)(zv —u?) + (gu ~ y2)(zu— yo) + (cv ~ yu)(yu— 22) = 0, (Wz,y,2,u,0); (*) Déi chiéu (I) va (*) ta dude he thite sau day (néu he (*) da cho c6 nghi M(af + be + cd) =0 ‘Ti d6 suy ra: 1° Hole A = 0, va do dé, he (*) c6 dang don gidn (I) sau: {az—y? ru yrs av yu=yu~2=2u-yo=z-w=0); (r) 2 Hoke » # 0 thi he (*) cling cdn nghigm khéc nifa khi cdc hing s6 da cho ring bude véi nhau béi diéu kign sau: af +be+cd = 0, (*) Vay ta cdn xét hai trutmg hop: Tru@ng hgp 1. Néu céc s6 thc khée khong a,b,c, d,e, f da cho Ia tay §, khong adi héi phai théa man digu kien (*) thi he phuong trinh (*) duce xét chi c6 nghiém tém 1BAl tofn 10** nay dé duge dua vio cubn séch ciia GS. Nguyn Vin Mau "Da thite dai s6 vA phin thie hau t7", NXB Gio due, 2002 (Bai toda 8, trang 48) 92 thudng, (ng v6i gid tri A = 0 trong (I). Nghiem d6 18 nghiém cia he phuong trinh (P) nh da chi ra 6 trén. Tit he (I) nay ta dé dang thiét lap duoc day diing thitc sau (1) ‘Tit (1) suy ra he (I’) nay o6 nghiem sau day: eryrziuivealiti? set, (véit tay y, tER); (2) Néu dé f ring 1 = 7? thi 2,y,z,u,v theo thi tu ty 1é v6i lay thita cia #, tir 0, 1,2,3 dén 4, Diu dé goi ¥ ring néu ta thay déi ky hiéu céc dn 56 x,y, z,u,v lan lugt bai %o)Z1, 22, 13, v4 thi biéu thttc nghiém cite he (I) duge viét gon lai dudi dang: a = pt’ (0 #0 bat ky, i € {0,1,2,3,4}) hay dac biét, 06 thé cho p = 1 thi: a =t (i € {0,1,2,3,4}); (3) ‘Trudng hyp 2. He H, duge bé sung them did kien (*) dé trd thanh he (H) = {(+), (+#)}. Tit day khong nhing chi thay déi ky higu d6i voi céc dn s6 nh trén ma cing thay ddi ky hiéu d6i y6i cdc hing s6 dé cho; cu thé 1A a,b,¢,d,e, f lan lugt duge thay bdi bo, bi, ba, bs, be, bs. Vide thay déi nay cling con c6 tac dung gitip chiing ta theo doi vige gidi he phuong trinh phan nao duge dé dang hon. Véi quy uée 46, ta viét lai he phuong trinh (1) = {(+),(##)} = (Me) U(#e) sau khi duge bd sung them didu kien (*): (% sect sagan = mais = BN _ aeons aed (_ \ 49) (*) bobs + biby + baby = 0 (+*) Con déng nhét thtic (*") 6 trén thi bay gid duge viét lai nhu sau: (Coa ~ a)(zamr4 — 23) + (oxy ~ 2129) (wary ~ 232) + (0x4 — 2129) (2425 - 23); (*”) 2) Tim nghigm khong tim thudng cita bai ton, ¢) Ngoai déng nhét thic (*”) lien quan dén céc dn s6 2; (i € {0,1,2,3, 4}) da duge phét hien trén, bing nhing quan sét dic biét céc nhj thttc thudn nhét cia céc An c6 cling bac (déng bac) va c6 cing téng céc chi s6, ta cn phét hien thém 5 déng nhét thitc nifa sau day (ma ban doc ciing dé dang kiém nghiem dugc): (war ~ 13)ot2 + (xoe3 — ay24)29 + (m2 ~ 23)x4 = 0, @ ~2(aare — 28)x1 — (wars ~ 2124) 02 + (core — 205 + 22)e9 —(aota — 2122)24 =0, (ii) (x24 ~ 23) 20 — (wots — 22%4)21 — 2(aor4 — © Z3)Z2 + (ots ~ 2122):t9 + (toma — 23)rq = 0, (ii) (2x03 — m24)z0 + (tots — 2m123 + 29)ay + (Zo ~ 2y2)2 — 2(z9z2 - 27)z3 = 0, (iv) (2129 — 23)a0 — (sors — 2122)21 + (or, — 2?) = 0. () 93 Bay gid ta thay cée nhj thifc 3 trong ngo&e béi nhj thite tuong tng 06 gi tri Ab, (= 0,1,2,3,4,5}) rit tir day ti s6 bling ohau (*) 6 trén, sau d6 khit dn phy ta thu duge mot he méi 1, gém 5 phuong trinh tuyén tinh thudn nhAt vdi 5 dn s6 @o, £1, L203, 24 ma dinh thite 4 cia he trigt tieu (vi A duge phan tich thinh tich hai thita s6, trong dé ‘mot thita $6 1a bobs + bibs + babs (= 0); thita s6 cdn lai, ky hiéu O(b,) # 0. beta + bets + bray = 0 (i 2b, - byte + (be —bs)ts - bite = 0 (ii), (11) { bs:t0 byt, - bytr + bits + bore = 0 (iii); bato + (bo—ba)tr + bray —2boxs 0 (iv), bsxo —bz, + dor, 0 (v)i Vi dinh thie A cia he (H:) o6 dang: A = (bobs + bibs + bubs) .6(b,) = 0 ma (0) £0 nén (741) 06 nghigm khong tam thudng duy nhét, (xéc dinh sai khéc mot thita s6 #0). Gidi he (74:) ta thu duge nghiém sau day: Auto = bf ~ bo(ba + bs), bobs + bibs, 2 — bobs, (4) = (bibs + babs), Aize = bj ~ bs(ba + bs). {é gidi he phuong trinh tuyén t{nh thudn nbét 1, nay (c6 dinh thie A = 0) ta cht vige 6 di mot trong 5 phuong trinh ciia he d6, ching han bé phuong trinh tht ba (iii); sau 6 chi vide gidi hé {74,\(iii),} gém 4 phuong trinh cdn lai véi 5 én Lo, Li, 22, 23,24 thi thu duge nghiém duy nhét c6 biéu thie (4)]. ‘Tom Iai la, sau hai bude gidi ta thu duge két qui He phuong trinh bac hai (1) {(),(#«)} v6i 5 dn zo,a1, 22,19, 24 06 nghiém tam thutng (3) vA nghiém kh6ng tam thudng (4) nhu da chi ra 6 tren, B. Co sé ly thuyét toan hoc din dén hé phuong trinh tuyén tinh thudn nhAt (74) dé tim nghigm khong tam thuong cia hé (71) néu trong bai todn 3. Ching ta cin thm hiéu ngon nginh dé ly gidi con dudug din dén Idi gii "so ef" ciia bai toén 10** nhu da néu ra trong myc A 6 trén. 1°) Trude hét, dé y rang cée hiéu sau day: ToRa— Tj, Bots ~ m2, Boe, Tyzg, tyTy~ 23, very — aia, yt — 22 Ia cée nhi thite bac hai thudn nhét eta céc dn x; (i € {0, 1,2, 3, 4}), xufit hién trong day (*) cée ty s6 bang nhau cia hé (H.), duge tao thanh tix ede dinh thitc con (minor) ofp 2 rét tit ma tran chit nhat (2x 4) sau: zo @% 22 25 jz) Zo 25 24 4 C6 thé xem chéing la céc toa dd Pliicker cli mot dudng thing xa anh trong khong gian xa dnb 3 chiéu P; xéc dinh béi hai diém 06 cfc toa d6: diém A(zo, 71, 22, ra) va diém B(x, 2,23,24) trong toa dé xa dnh thudn nbét. 2) Clic ¥ tudng co bin trong qué trinh tim t0i di gidi (nghiém) ctia hé phutong trinh (H) 6 tren. a) Nhu d phan A da chi ra: Hé phuong trinh 71, (*) va do dé, hé phuong trinh (1) 06 nghiém tm thutng ma biéu thitc nghiém c6 dang (3) a= pt, 4 {0,1,2,3,4}, p #0. b) Hé phuong trinh 11, (*) v6i cde dn 2; duoc viét Iai nhwt sau: ry bo = tote — 2}, phy = tomy — 2122, pho = zox4 — 2125, pbs = 103 — 03, py = t9%3— 2174, pbs = xatq — 23; trong 6 p la gid tri chung cita 5 ti s6 bling nhau ofa hé phuong trinh (*) ma gié tri p= 0 cho ta nghi¢m tam thudng (3) cia hé (71) va chua doi héi dén diéu kign (**) cia céc hing s6 cho trudc b; (i € {0, 1,2, 3,4, 5}). Ciing nhu nhan xét b) phin A da chi ra: Voi ao, 21, 22,29, 24 tuy ¥ bao gid ta cing 6 déng nhat thite (*’): Way (i = 0,1, 2,---74) : (woe — 29) (wae — 23) + (toa — 2109)(973 — 2424)+ + (Gots — 2123)(m123 — 23) =0; (*) Chink déng nhAt thie (*’) nay cing v6i ding thite diéu kign (**) vé céc hling 36 by (i= 0,1,-758) la co sé quyét dinh cho vige tim nghigm khéng tém thuing cia he (2). c) Ta chimg minh didu khing dinh sau: ‘Néu 06 he thitc (**) nghia 1a cdc hing s6 b; (i = 0,1,...,5) rang bude béi digu kién bobs + bibs + babs = 0 thi, ngodi nghi¢m tam thutng (3) (x; = ¢ (i € {0,1,2,3,4})], he phwong trinh bac hai thudn nhét (71) = {(4), (++)} con o6 mot nghiém khong tam thudng duy nhét ma (4) 1& biéu thie nghigm da chi ra 4 phan A. ‘That vay, hé thitc (**) duge viét lai duéi dang tuong duong sau: 2p\bobs + bibs + babs) = 0 (voi p #0), hay la bo(pbs) + bs (abo) + bx (abs) + bs(0bs) + ba(pbs) + Ba (pbs) = 0. @) Thay céc gié tri ctia pb; (i = 0,1,-.-,8) tit (1") vao (*") ta dude he thie sau day (sau khi da nhan hai vé cia (*”) véi 2): 2lbo(zaxa — 23) + bo(ore ~ af) + bi(z2xq — 2124) + by zor — 2122)+ + bo(mizs — 23) + ba(tors — 2129) = 0. (I") 95 V6i céc by (¢ = 0,T,..., 5) 1a da cho thi (I") chinh la phuong trinh trong toa do xa anh thudn nhét cia mét si¢u mat bac hai, suy bién think mot siéu nén trong khong gian xa. anh 4 chiéu P, Bay gid ta viét lai phuong trinh (I") dudi deng ma tran 27Qax. 0: 4 Dawes = 0, (gy = ait) ©) ijno trong dé cfc hé sé qi; c6 cdc gid tri cu thé nhu sau: Gos = bs, 13 = be — bs, Gs = bf, (6) G33 = —2bp, dua = 0, ‘Vay duéi dang ma tran thi phyong trinh siéu quadric (5) c6.dang: 0 0b & b\ fae 0-2; —by b—bs | fr (Zo 1 2 25 t4)] bs by -2% b bo | |x] =0, (6) by b2—bs by = 2b 0 ts bs bh by) Ny hay viét (6) mot cach nggin gon hon, duéi dang toén tit: 2™Qz =0, Ve #0 = (0,0,0,0,0) (7) trong 46: x7 = (a9, 21,2, 5,74) IA véct hang (chuyén vj clia véct cot (x9, -..,24), va Q = Q(b) 18 mot ma tran vuéng cp 5, c6 cée phan ti gy tao thanh ttt cdc hing s6 5; d& cho (théa man (**)) theo bang (5') 6 trén), Sau khi thye hign phép tinh dinh thiic |Q| cha ma trén Q(b) trong 46 } = (bo; bi, bays, bay bs) thi duge: |Q(b)| = (bobs + bibs + babs).-O(bi) = 0 (vi theo (**) thi bobs ++ bibs + babs = 0), nghia 18 Q(b) 1& mot ma tran suy bién va c6 hang r = 4. Boi vay, (6) 1a phuong trinh cia mot siéu mat baé hai suy bién thanh mot siéu ndn bac hai 6 O-phing_ dink, tite phing-dinh nay 1a mot diém, dugc xéc dinh boi hé phutong trinh tuyén tinh thudn nhét sau: Dui dang toén til, he phuong trinh nly duge viét gon lai Ia: Qzr=0, (8) Vi ma tran vudng cép 5 Q cé hang r bang 4 nén trong 5 phuong trinh tuyén tinh thuin nhét cia hé (8) chi c6 4 phuong trinh la doc lap tuyén tinh. Boi vay, hé phuong trinh tuyén tinh thudn nhét (8) xéc dinh cho ta mét diém duy nhdt trong khong gian xa anh. bén chiéu Pj. Diém ndy chinh 1a dink ctia sieu nén bac hai cb phutong trinh (6) hot (7) trong khong gian xa anh P, nay, 96

You might also like